MINISTRY OF PUBLIC HEALTH OF UKRAINE

Department of human resources policy, education and science

Testing Board

Student ID Last name

Variant ______

Test items for licensing examination

Krok 2

STOMATOLOGY

General Instruction

Every one of these numbered questions or unfinished statements in

this chapter corresponds to answers or statements endings. Choose the answer (finished statements) that fits best and fill in the circle with the corresponding Latin letter on the answer sheet.

ББК 54.1я73 УДК 61

Authors of items: Aleksandrov O.O., Andrianova I.I., Artyukh V.M., Belaiev E.V., Bondalyetov V.O., Bondarenko V.S., Bosa L.F., Byelikov O.B., Cheliy O.I., Chernov D.V., Chernov O.YE., Chygrynets V.N., Denisenko O.G., Derkach L.Z., Devdera O.I., Dmytriyeva A.A., Dubova L.I., Dzhereliy A.O., Feshchenko I.F., Filimonov YU.V., Forostyaniy S.A., Furdichko A.I., Furman R.L., Gaiduk R.V., Gerasimchuk P.G., Golik V.P., Golovko N.V., Golubyeva I.M., Gordon-Zhura G.S., Gorlenko O.V., Grekulyak V.V., Grynovets’ B.S., Gubanova O.I., Gumets’kiy R.I., Gurzhiy O.V., Idashkina N.G., Ilnyckiy YA.M., Isakova N.M., Ivchenko N.A., Kaladze K.M., Karelina L.S., Kas’kova L.F., Kharchenko O.I., Kharkov L.V., Kl’omin V.A., Kolesnichenko O.V., Kolesnyk K.A., Kolesnyk V.M., Kolomeichuk B.YA., Kordys M.S., Kosaryeva L.I., Kostyshyn Z.T., Kovalenko M.O., Koval’ O.V., Koval’ S.M., Koval’chuk L.P., Kovach I.V., Kozynenko L.A., Krupnik N.M., Kuznecova L.V., Kuz’ G.M., Kurilets’ KH.S., Loganovs’ka YE.M., Lomeks O.I., Lungu V.I., Lysenko YU.G., Makarevich A.YU., Matviychuk O.YA., Mazuryna I.O., Mirchuk B.M., Mironova I.V., Mizyuk L.V., Moiseitseva L.O., Morozova M.M., Muzychina G.A., Neiko N.V., Nesin O.F., Novikov V.M., Odzhubeiska O.D., Oleksandrov O.O., Ovcharenko E.N., Ozhogan Z.R., Pan’kevych A.I., Polishchuk S.S., Popova O.I., Prots’ G.B., Potiyko V.I., Raiyda A.I., Rimsha O.YE., Riznyk S.S., Romanenko I.G., Romanovs’ka A.P., Romashkina O.A., Rubizova A.G., Samsonov O.V., Satanovskiy M.A., Scherbyna I.M., Shapovalov A.S., Sharan M.O., Shuklyn V.A., Shutak O.V., Sklyar S.O., Strel’chenya T.M., Svirchkov V.N., Sydorenko I.V., Sylenko YU.I., Syriy O.M., Tereshyna Z.L., Tsentilo V.G., Tsysynskiy M.B., Tyugashina YE.G., Udod O.O., Val'chuk O.G., Volovar O.S., Yeroshenko O.A., Zamureyko A.I., Zelinc’ka N.A., Zhdanov V.YE. and committees of professional expertise.

Item reviewers. Byelikov O.B., Gerelyuk V.I., Zhegulovich Z.E., Ilenko N.M., Kaskova L.F., Kosareva L.I., Kril A.Y., Moiseytseva L.O., Muzichina G.A., Muntyan L.M., Nerodenko N.I., Nesin O.F., Nikolishin A.K., Ostapko O.I., Rayda A.I., Rozkolupa O.O., Samsonov O.V., Sedletska A.O., Solonko G.M., Tril S.I., Usenko S.A., Filimonov YU.V., Kharchenko O.I., Yakovenko L.M.

The book includes test items for use at licensing integrated examination “Krok 2. Stomatology” and further use in teaching.

The book has been developed for students of stomatological faculties and academic staff of higher medical educational establishments.

Approved by Ministry of Public Health of Ukraine as examination and teaching publication based on expert conclusions (Orders of MPH of Ukraine of 14.08.1998 №251, of 27.12.1999 №303, of 16.10.2002 №374, of 29.05.2003 №233).

© Copyright Testing Board. Крок 2 Stomatology (англомовний варiант, iноземнi студенти) 2015 рiк 1

1. After examination a 5-year-old child was accident. The patient stayed conscious. He diagnosed with acute general pulpitis of the 74 complains of headache, dizziness, general tooth. What treatment will be most efficient in weakness, nausea. 12 hours after the injury this case? the patient got "raccoon eyes"(periorbital haematomas) within the limits of the circular A. Pulp extirpation muscle of eye. What fracture does the victim B. Vital amputation have? C. Non-vital amputation D. Biological method A. Fracture of skull base E. Tooth extraction B. Le Fort II fracture of maxilla C. Fracture of nasal bones 2. A 55-year-old female patient complains D. Bilateral fracture of zygomatic bones of pain during eating food, erosions coveri- E. Fracture of frontal bones ng . The disease onset was more than one month ago. Objectively: the mucosa 7. In the process of root extraction of the 38th of the , soft and dental arches is tooth with Lecluse root elevator a dentist has unchanged, but has large bright-red erosions. pushed the root through into the perimandi- The mucosa is easily damaged and exfoliates bular soft tissues. The root is situated under when rubbed. The pain is slight. Impression the alveolar arch mucosa and can be clearly smears contain Tzanck cells. What is the most palpated. What approach should the doctor probable diagnosis? choose in the given case? A. vulgaris A. Dissection of soft tissues covering the root B. Lever’s bullous pemphigoid followed by root extraction C. Benign nonacantholytic pemphigus B. To try and extract the root through the tooth D. Erythema multiforme socket E. Lichen ruber planus, bullous form C. Surgical root extraction through extraoral submandibular way of access 3. A 17-year-old female patient complains of D. Further actions can be determined only after gingival growth in the area of the upper and X-ray examination lower front teeth. When examined, the gingi- E. To leave the root in the soft tissues val papilla appear to be hyperemic, swollen, cover the teeth crowns up to 1/2, bleed when 8. An 8-year-old girl complains of bleeding touched. Pathology of the following system is gums. The child suffers from pancreatic di- important in etiology of this disease: abetes. Objectively: gingival papillas in the area of the upper and lower jaw teeth are A. Endocrine hyperemic, swollen, cover 1/3 of the crown. B. Cardio-vascular What tests are necessary to determine severity C. Digestive of the illness? D. Immune E. Nervous A. X-ray radiography B. Reoparodontography (Impedance 4. A 48-year-old male patient complains of itching gums. Objectively: the gums are firm; phlebography of ) cold water causes short-time pain in the teeth. C. Capillary fragility test X-ray imaging shows osteosclerosis of the D. PMA index alveolar septum: close-meshed bone structure, E. Stomatoscopy the alveolar septum height and intactness of 9. A 47-year-old female patient complains of the cortical plates is retained. What is the most inability to eat hot and cold food, as well probable diagnosis? as of intense pain caused by sour, sweet and A. Initial parodontosis salty food. Objectively: there is a slight loss B. Atrophic of enamel on the molars and incisors. Probi- C. Initial periodontitis ng and cold test cause acute pain. What is the D. Periodontitis, stage I most likely diagnosis? E. Parodontosis, stage I A. Hyperesthesia of dental hard tissues 5. A patient complains of spontaneous, B. Enamel erosion paroxysmal, irradiating pain with short pain- C. Enamel necrosis free intervals. The pain arose 2 days ago and D. Enamel hypoplasia occurs only at night. Make a provisional di- E. Pathological abrasion of dental hard tissues agnosis: 10. A 36-year-old patient complains of pain A. Acute diffuse pulpitis under the dental bridge. After its removal the B. Acute deep caries patient has been found to have an ulcer 0,3x0,5 C. Exacerbation of chronic periodontitis cm large on the alveolar process. The ulcer D. Acute circumscribed pulpitis is slightly painful and soft, the surrounding E. Acute purulent pulpitis mucosa is hyperaemic, submandibular lymph nodes are not enlarged. What is a provisional 6. A 34-year-old patient had got in a car diagnosis? Крок 2 Stomatology (англомовний варiант, iноземнi студенти) 2015 рiк 2

defect: the placement of the 23rd tooth is A. Decubital ulcer incorrect. Objectively: the face is symmetrical. B. Trophic ulcer The 24th tooth has buccal placement above the C. Sutton aphtha occlusion area. The space between the 22nd D. Cancerous ulcer and 24th is 3 mm. What additional examinati- E. Tuberculous ulcer on is required? 11. A 45-year-old female patient presents to a A. All of the methods named below dental clinic for prosthetics. Objectively: the B. Pont analysis 47, 46, 45, 35, 36, 37 teeth are missing. There C. X-ray examination is dentoalveolar vertical displacement of the D. Measuring the length of the dentititon 17, 16, 26, 27 teeth; the alveolar process is E. Korkhaus analysis hypertrophied, the cervical margins are not exposed. When the teeth are closed, the di- 16. A 48-year-old female patient had been deli- stance between the occlusal surfaces of teeth vered to a hospital with a wound and and the alveolar process is about 6 mm. Speci- severe headache, nausea, dizziness. It is known fy the most efficient method of treatment of from the history that the patient had got a secondary deformation in this patient: trauma as a result of a fall. After examination she was diagnosed with a contused lacerated A. Pulp removal and teeth grinding wound of cheek, closed craniocerebral injury, B. Surgical treatment brain concussion. This trauma can be qualified C. Grinding as: D. Hardware-associated surgical treatment E. Disocclusion A. Concominant trauma B. Isolated trauma 12. A 58-year-old patient has made an appoi- C. Combined trauma ntment to make a denture. Objectively: the D. Polytrauma 22nd tooth is stable and intact. The alveolar E. Single crest is atrophied; palate is flat. Removable denture is to be made. What approach regardi- 17. Parents of a 6-year-old child are concerned ng the 22nd tooth should be chosen by a denti- about their child refusing to eat and havi- st? ng grey-and-yellow coating in the oral cavi- ty. The symptoms are observed over a peri- A. To make a telescopic crown od of one year. The child complains of sharp B. To retain the tooth dryness and burning pain in the mouth. C. Removal of tooth pulp The anamnesis states chronic bronchitis D. Tooth extraction and systematic treatment using anitibiotics. E. To make a stump crown Objectively: swollen and hyperemic mucous membranes of and are covered in 13. A 35-year-old patient has made an appoi- yellow and pale-grey coating. There is coati- ntment with a doctor to have oral cavity sanati- ng on the , which is connected with the on. Objectively: the vestibular surface of the tissue, cannot be removed and has infiltration 22nd tooth has a hard tissue defect localised basis. What is the most probable diagnosis? in the mantle dentin; the dentin is hard, dark- coloured, the floor of the cavity is coarse. Cold A. Chronic candidiasis of mouth stimulus and probing are painless. What is the B. Acute herpetic most probable diagnosis? C. Chronic recurrent D. Erythema multiforme A. Chronic median caries E. Acute candidiasis of mouth B. Enamel erosion C. Cuneiform defect 18. A 24-year-old female patient has Ri- D. Dental fluorosis, destructive form chmond crown being made to restore the E. Hard tissue necrosis crown of the central maxillar incisor. The cap is completed. What step should be the next? 14. A 12-year-old boy complains of a cavity in a tooth on the lower left jaw. Objectively: 1/3 of A. To fit the cap on the tooth stump and place the 36th tooth is destroyed, the carious cavity the post in the root canal opens into the dental cavity; there is sensitivity B. To solder the post with the cap to cold stimulus; probing and percussion are C. To fit the cap and the post to the tooth root painless. X-ray imaging shows the periodontal D. Making of combination dental crown fissure of the 36th tooth roots to be widened. E. Tooth fixation with cement What is the provisional diagnosis? 19. A patient complaining of constant dull pain A. Chronic fibrous periodontitis in the 38th tooth has made an appointment wi- B. Chronic simple pulpitis th a dentist. Mouth opening is restricted to 1 C. Chronic gangrenous pulpitis sm. The patient is prescribed the extraction D. Chronic granulating periodontitis of the 38th tooth. What kind of anaesthesia E. Chronic granulomatous periodontitis should be administered? 15. An 11-year-old girl complains of a cosmetic Крок 2 Stomatology (англомовний варiант, iноземнi студенти) 2015 рiк 3

A. Веrcher-Dubov anaesthesia A. Chronic fibrous pulpitis B. Mandibular B. Exacerbation of chronic pulpitis C. Torusal C. Acute deep caries D. Extraoral administration of mandibular D. Chronic deep caries anaesthesia E. Chronic fibrous periodontitis E. Tuberal 24. A 32-year-old male patient complains of 20. A 67-year-old patient complains of dryness and burning pain in the back of the recurrent erosion on the red border of the tongue. The symptoms have been lasting for lower . Objectively: the erosion is oval 1 week. The pain increases when spicy food in shape, 0,8х1,3 sm in size, covered in thin is taken. A day prior to that the patient had scabs that reveal glossy surface with punctate suffered from pneumonia and had been taking bleeding when removed. There are atrophic antibiotics. Objectively: the skin and mucosa areas of the red border detected. Infiltrati- of oral cavity are pale. The tongue mucosa on elements are absent. The submandibular is hyperemic and swollen; there is crumbling lymph nodes are not enlarged. What diagnosis grey-and-white coating in the folds of the back can be assumed? of the tongue; the lateral surfaces of the tongue are desquamated. The saliva is thick and drags A. Manganotti’s abrasive precancerous in threads after a spatula. Choose the most B. , erosive ulcerative form effective drug complex for topical treatment. C. Keratoacanthosis D. Bowen’s disease A. Methylene blue + Pimafucin (Natamycin) E. Cheilitis glandularis B. Hydrocortisone ointment + Solcoseryl C. Decaminum (Dequalinium chloride) + 21. A 66-year-old patient complains of dense Hydrocortisone raised induration at the tip of the nose. D. Furacilinum (Nitrofural) + Methylene blue Protruding part of the induration often breaks off and then continues to grow. Objectively: E. Furacilinum (Nitrofural) + Solcoseryl there is an affected area with focus up to 1 25. A 37-year-old woman has made an appoi- sm in diameter at the tip of the nose. There ntment to extraxt the 25th tooth. What kind of is a protrusion growing from the focus; the anaesthesia is the most advisable? protrusion is 1-1,2 sm high, brownish-grey in colour, with dense consistency. What diagnosis A. Unilateral tuberal, infraorbital and palatinal can be assumed? anaesthesia B. Unilateral tuberal and palatinal anaesthesia A. Cutaneous horn C. Unilateral infiltration anaesthesia from the B. Keratoacanthoma vestibular surface C. Rhinophyma D. Unilateral infraorbital and incisoral D. Fibroma anaesthesia E. Hard papilloma E. Unilateral infraorbital and palatinal 22. A 53-year-old patient complains of pain anaesthesia and clicking in the left temporomandibular 26. A 70-year-old patient with all his lower jaw joint. Objectively: the face is symmetrical, teeth lost is undergoing the process of fitti- palpation of the lateral pterygoid muscles is ng individual dental spoon using Herbst tests. painful on the left side. Mouth opening is When the mouth is opened slowly the spoon restricted. Tomogram shows the bone outline rises in the front area. What area requires of joint surfaces to be smooth. Which disease correction of the spoon placement? of those given below corresponds with this cli- nical presentation? A. Vestibular surface between canines B. Behind the retromolar trigone to mandi- A. Temporomandibular joint disfunction bulohyoid line B. Rheumatic arthritis C. Frenulum of tongue area C. Deforming arthrosis D. From the retromolar trigone to the place, D. Acute posttraumatic arthritis where the 2nd molar will be placed E. Joint ankylosis E. 1 cm from the midline on the sublingual end 23. A 38-year-old male patient complains 27. Preventive examination of a 4,5-year- of carious cavity. Prior to that there were old child revealed some hidden cavities on attacks of toothache with no external cause. the contact surfaces of the 54 and 55 teeth. Objectively: the distal masticatory surface of After removal of the overhanging edges of the 37th tooth has deep carious cavity filled the enamel the softened dentin could be with softened pigmented dentine. Percussi- easily removed with an excavator within the on is painless. Upon the extraction of the mantle dentin. Select the optimal material for degenerated tissue cold water stimulus causes a permanent filling: pain lasting about 1 minute. X-ray imaging shows deformation of the periodontal fissure in the area of the root apexes of the 37th tooth. What is the most probable diagnosis? Крок 2 Stomatology (англомовний варiант, iноземнi студенти) 2015 рiк 4

A. Compomer material A. Vankevych dental-gingival splint B. Composite material B. Port gingival splint C. Silicate cement C. Gunning gingival splint D. Silicophosphate cement D. Weber dental-gingival splint E. Polycarboxylate cement E. Zbarzh appliance 28. A 38-year-old female patient complains of 32. A 22-year-old patient complains of a pai- pain in front of the external auditory meatus nful swelling in the right . A week projection, clicking when opening the mouth, earlier the patient got a cheek abrasion which sensation of stuffiness in the ears. Objectively: healed under the purulent crust. Over the past the face is symmetric, the trajectory of mouth two days the patient had observed progressi- opening is straight. There is a I class Kennedy ng pain and fever up to 38, 6oC. Objectively: defect, the 18, 17,16, 26, 27,28 teeth are missing. there is a soft tissue edema in the right parotid Which anatomical structure takes the greatest region, the skin is slightly strained but has not load in this situation? changed in colour. There is a dense painful infiltration 2,5x3,5 cm large, the skin over it A. Articular disc (meniscus) exhibits limited mobility. The mouth can be B. Articular head fully opened, the around C. Distal surface of the medial articular tubercle the orifice of the salivary duct is unchanged, D. Bottom of the glenoid fossa of the temporal the saliva is transparent. What is the most li- bone kely diagnosis? E. Articular capsule A. Acute lymphadenitis 29. An 11-year-old child complains of pain B. Exacerbation of chronic and bleeding gums when eating, bad breath, C. Abscess of the parotid-masseteric region weakness and headache that have been lasting D. Acute non-epidemic parotitis for two days. Objectively: gingival mucosa in E. Epidemic parotitis the area of front lower teeth is swollen, brightly hyperemic, easily bleeds when touched. Gingi- 33. A 44-year-old patient complains of painful val papillas are covered with dirty-grey coati- and restricted mouth closing, inability to take ng; when coating is removed the underlaying food and sharp pain in the temporomandibular surface is bleeding and painful. The occlusion joint. Objectively: the mouth is half-open, the is deep. There are significant deposits of soft lower jaw is displaced in the distal projecti- dental plaque. The regional lymph nodes are on, the lower jaw processes are displaced slightly enlarged and painful when palpated. upwards; of the masticatory muscles Body temperature is 37, 6oC. What is the and swelling in the temporomandibular joint provisional diagnosis? areas are observed. What is the most probable diagnosis? A. Acute ulcerative gingivitis B. Acute catarrhal gingivitis A. Dystal mandibular dislocation C. Chronic catarrhal gingivitis B. Bilateral mandibular fracture in the mandi- D. Chronic hypertrophic gingivitis bular angle area E. Generalized periodontitis C. Acute arthritis of temporomandibular joints D. Bilateral ankylosis of temporomandibular 30. An 18-year-old student needs prosthetic joint porcelain-fused-to-metal denture for the 11, E. Bilateral fractures of the condylar processes 21 teeth. There are no contraindications for of mandible the use of such construction. What is the most appropriate material for taking impressions? 34. A 58-year-old patient has significantly narrowed mouth opening due to trauma and A. Sielast burns of the face. He is prescribed partial B. Stomalgin removable denture. What denture constructi- C. Orthocor on is the most advisable in the given case? D. Stens E. Repin A. Folding denture B. Clammer-stabilized clasp (bugel) prosthesis 31. A 67-year-old patient complains of pain, C. Attachment-stabilized clasp (bugel) dental bleeding, chewing disorder. The anamnesis prosthesis states domestic injuri of the lower jaw. Objecti- D. Jointed removable denture vely: the upper jaw dentition is retained. The E. Simple removable partial laminar denture lower jaw has no teeth, there is central fracture of the lower jaw with bone defect, the place 35. A 40-year-old patient complains of pain in of the fracture is swollen. What construction is the lower jaw, malocclusion. He sustained a the most advisable in this case? trauma 6 hours ago. Examination revealed a singular lower jaw fracture between the second incisor and the left canine. Fragments di- splacement is insignificant. All teeth are retai- ned and stable. What is the optimal treatment? Крок 2 Stomatology (англомовний варiант, iноземнi студенти) 2015 рiк 5

A. One arch dental braces Zbarzh set. Which construction provides the B. Osteosynthesis with mini plate intra-oral fixation? C. Full dental braces D. Gartsatnikov splint A. Standard double arch E. Vasilyev splint B. Aluminium arch bar C. Plastic biteplate 36. Having recovered from acute respiratory D. Weber’s splint disease, a patient has made an appointment E. Crown-supported soldered splint with a dentist. The patient complains of pain in the gums, bad breath, inability to take food, 41. A 32-year-old patient has made an appoi- general weakness. Objectively: the gums are ntment with a dental surgeon to have oral hyperemic, swollen, covered in necrotic dirty- cavity sanation performed prior to denture grey coating; the gums bleed when the coati- installation. During examination the doctor ng is removed. Microbiological study of ti- stated the following: the crown of the 25th ssues revealed a great number of cocci, bacilli, tooth is destroyed. The tooth root is stable, and fusobacteria, and spirochaete. Specify the drug its percussion is painless. Mucosa of alveolar for etiotropic treatment. process is unchanged. X-ray imaging reveals slight widening of periodontal fissure. What A. Metronidazole provisional diagnosis can be made? B. Galascorbinum C. Potassium permanganate A. Chronic fibrous periodontitis of the 25th D. Tripsin tooth E. Carotolinum (Betacarotene) B. Chronic periodontitis of the 25th tooth C. Chronic granulomatous periodontitis of the 37. A 4-year-old boy has been diagnosed wi- 25th tooth th acute purulent periostitis of the upper D. Chronic granulating periodontitis of the 25th jaw from the 64th tooth. Choose the correct tooth approach in treatment. E. Cystogranuloma A. The 64th tooth extraction, periosteotomy, 42. A 10-year-old girl complains of an aesthetic pharmacotherapy flaw. The anamnesis states, that she had B. The 64th tooth extraction, anti-inflammatory been sucking her right thumb up to the pharmacotherapy age of 7. Objectively: the face lower thi- C. Endodontological treatment of the 64th rd is somewhat reduced. The sagittal fissure tooth, anti-inflammatory pharmacotherapy between the upper and lower incisors, is 9 D. Endodontological treatment of the 64th mm wide, class 2 according to the Angle tooth, periosteotomy classification. Eshler-Bittner test leads to ini- E. Periosteotomy, anti-inflammatory tial temporary improvement of the girl’s face, pharmacotherapy followed by renewed deterioration. What cli- nical malocclusion is the most probable in this 38. During the examinations and everyday case? orthopaedic manipulations a dentist uses a dental mirror. What is the way of its sterili- A. Maxillary macrognathia and mandibular zation? micrognathia B. Maxillary macrognathia A. In the triple solution for 30 minutes C. Mandibular micrognathia B. In the 0,5% ethyl chloride solution for 20 D. Maxillary prognathism with lateral minutes compression C. In the 6% hydrogen peroxide solution for 6 E. Mandibular retrognathia hours D. In the dry heat sterilizer at 180oC for 10 43. A 35-year-old patient complains about itch, minutes burning and edema of lips. These presentati- E. In the 0,01% chloramine solution for 10 ons occured a week ago. Objectively: there minutes is reddening of and skin, especially in the region of mouth corners, there 39. A 45-year-old patient has central defect of are also vesicles, crusts, small cracks along wi- the . The defect is 2х3 sm in si- th erythematous affection of vermilion border. ze. Dentition is intact. What construction of What is the most likely diagnosis? obturator is the the most advisable in this case? A. Acute eczematous cheilitis A. Palatal obturator B. Multiform exudative erythema B. Pomerantseva-Urbanskaya obturator C. Acute herpetic cheilitis C. Floating obturator D. Allergic contact cheilitis D. Ilyina-Markosyan obturator E. Exudative form of exfoliative cheilitis E. Laminar denture with obturating element 44. A 44-year-old male patient complains 40. A 32-year-old patient presented to a of fatigue and headache, limb numbness, prosthetic dentistry clinic with a diagnosis of dry mouth, burning and pain in the tongue. maxillary fracture.The fracture can be treated Objectively: skin and oral mucosa are pale. by means of standard maxillo-mandibular There are painful cracks in the corners of Крок 2 Stomatology (англомовний варiант, iноземнi студенти) 2015 рiк 6 mouth. Dorsum of tongue is smooth, glossy, A. Injection with bright red striae. Blood count: Hb- 70 B. Audioanalgesia g/l, RBCs - 1, 5 · 1012/l, colour index - 1,6, C. Topical leukopenia, thrombocytopenia, lymphocytosis. D. General anesthesia What is the most likely diagnosis? E. Cryanesthesia A. Addison-Biermer anemia 49. An 8-year-old child needs his 74th tooth B. Chronic posthaemorrhagic anemia extracted due to exacerbation of chronic peri- C. Late chlorosis odontitis. What kind of anaesthesia should be D. Iron deficiency anemia used? E. Aplastic anemia A. Mandibular anaesthesia 45. A 35-year-old male patient is prescribed B. Torusal anaesthesia adhesive dental bridge. Objectively: the 15th C. Infiltration anaesthesia tooth is lost. The 14th and 16th abutment D. Application anaesthesia teeth are stable. The teeth crowns are of medi- E. Mental nerve block um height. The teeth equator is pronounced. 50. A 25-year-old patient complains of pain Occlusion is orthognathic. What area of the caused by eating sweet, hot and cold food; abutment teeth MUST NOT be covered with pain ceases, when stimulation stops. Objecti- denture framework? vely: the adjoining surface of the 36th tooth A. Cervical circumference of the teeth has a deep carious cavity localised in the ci- B. Tooth equators of the approximal surfaces rcumpulpar dentin. The dentin is softened. C. Tooth equators of the oral surfaces Probing of the carious cavity floor is painful. D. Fissures of masticatory surfaces What is the most probable diagnosis? E. Below the equator of vestibular surfaces A. Acute deep caries 46. When a prosthodontist was preparing the B. Acute median caries patient’s tooth, the patient had epileptic sei- C. Chronic fibrous pulpitis zure that was subsequently terminated. What D. Chronic median caries mistake had been made by the doctor? E. Chronic deep caries A. No inquire into the patient anamnesis 51. A 24-year-old patient complains of aching B. No inquire into the antecedent anamnesis pain in the 11 tooth that is getting worse during C. No anaesthesia biting down on food. Two days ago the tooth D. Crude preparation was filled for pulpitis. Objectively: the 11 tooth E. Did not decline the appointment is filled. The thermal test causes no pain, verti- cal percussion is slightly painful. X-ray picture 47. A 30-year-old patient complains of of the 11 tooth shows that the endodontic filli- toothache caused by hot and cold stimuli. ng is 1 mm above the root apex. Which of the The pain irradiates to the ear and temple. following methods will be most effective for Previously there was spontaneous nocturnal eliminating this complication? toothache. Objectively: on the occlusal surface of the 37 tooth there is a deep carious cavi- A. Fluctuorization ty communicating at one point with the tooth B. Ultrahigh frequency therapy cavity. Probing at the communication point, as C. Relaxing incision well as cold stimulus, cause acute pain. The D. Submucous injection of 1% solution of pain persists for a long time. Electric pulp test hydrocortisone result is 55 mA. What is the most likely di- E. Analgetics agnosis? 52. A removable denture is being made for a A. Exacerbation of chronic pulpitis patient. At the stage of designinig the denture B. Acute diffuse pulpitis accessory material - isocol - is used. What C. Exacerbation of chronic periodontitis group does it belong to? D. Chronic concrementous pulpitis A. Insulation E. Acute purulent pulpitis B. Impression 48. A 34-year-old-patient complains of hard ti- C. Modeling ssues defect of the 21st and 22nd teeth. He is D. Forming referred for porcelain-fused-to-metal crowns E. Polishing installation for his 21st and 22nd teeth wi- 53. A 46-year-old patient, after his recovery thout removal of tooth pulp. What kind of from flu, has suddenly started suffering from anaesthesia administration should be used in pain in the 36th tooth, the crown of which had tooth preparation? been destroyed. The soft tissues surrounding the lower jaw are swollen. There are high fever up to 39oC and chill observed. Objectively: there is prominent swelling of the left lower jaw alveolar mucosa (both oral and vestibular surfaces). The 34th, 35th and 37th intact teeth Крок 2 Stomatology (англомовний варiант, iноземнi студенти) 2015 рiк 7 are mobile. Percussion is painful. The gingival anterior palatal bars, tonsils. The submandi- pockets produce purulent exudate. Vincent’s bular lymph nodes are slightly enlarged, pai- symptom is observed. What diagnosis can be nless. Name the causative agent of this disease. assumed? A. Coxsackie virus A. Acute odontogenic mandibular osteomyeli- B. virus tis starting from the 36th tooth C. Epstein-Barr virus B. Exacerbation of chronic periodontitis of the D. Klebs-Loeffler bacillus 36th tooth E. Bordet-Gengou bacillus C. Acute suppurative mandibular periostitis starting from the 36th tooth 58. A 12-year-old girl has complaint of carious D. Periodontitis cavity in a tooth. Objectively: there is class 1 E. Exacerbation of chronic mandibular carious cavity according to the Black’s classi- osteomyelitis fication in the 36th tooth; it is localised in the parapulpar dentin; the mouth of the cavity is 54. A doctor treating an AIDS-positive patient wide. The dentin is dense and pigmented. It has accidentally pierced his own skin of the is sensitive to cold stimulus, percussion is pai- palm with a root needle. What should be the nless. What is the most probable diagnosis? first course of actions? A. Chronic deep caries A. Squeeze out blood, process the breach with B. Chronic median caries 70% alcohol solution C. Acute deep caries B. Squeeze out blood, process the breach with D. Acute median caries strong solution of potassium permanganate E. - C. Apply tourniquet to the shoulder D. Process the breach with 5% iodine solution 59. A 35-year-old female patient has complai- E. Process the breach with 3% hydrogen nts of cosmetic defects of the front upper peroxide solution teeth crowns. The defects have been increasi- ng for the last 10 years. The patient suffers 55. A 45-year-old patient complains of mi- from unpleasant sensations when brushing her ssing teeth in the lower jaw on the right. teeth, and when chemical stimuli are appli- Objectively: the 46, 45, 38, 48 teeth are mi- ed. Objective examination revealed defects ssing. The 47 tooth is mobile (1 grade mobi- localised in the enamel of the front upper teeth lity), the crown of the 44 tooth is decayed vestibular surface. The defects are oval, saucer- by 1/2. What orthopedic construction should shaped, and have clear margins. Probing and be recommended for restoration of masti- cold stimuli sensitivity test resulted in positive catory efficiency and prevention of periodontal reaction. Make the diagnosis. overload of the 47,44 teeth? A. Enamel erosion A. Bugel denture with clasp fixation for the 47, B. Enamel hypoplasia 44, 34 teeth C. Cuneiform defect B. Soldered bridge supported by the 48, 44 D. Chemical necrosis of the tooth teeth E. Hyperesthesia of tooth hard tissues C. Porcelain-fused-to-metal bridge supported by the 47,46 teeth 60. A prosthodontist uses a mouth mirror for D. Bridge supported by the 47 tooth examination and manipulation during regular E. Small saddle denture with clasp fixation for appointments. How are mouth mirrors sterili- the 47,44 teeth zed? 56. A 53-year-old female patient complains A. With 6% hydrogen solution freshly prepared on the day of use for 6 hours of feeling of tightness and coarseness in the o mucosa, the red border and the skin of the face. B. In a dry-heat sterilizer under 180 Celsius for Objectively: the skin of the face has butterfly- 10 minutes shaped damage, the red border is infiltrated C. With 1% chloramine solution for 10 minutes and tightly covered in hyperkeratinized scales. D. With 0,5% sulfachlorantin solution for 20 When an attempt is made to remove them, minutes they bleed and are painful. What disease is this E. With triple solution for 30 minutes clinical presentation characteristic of? 61. A 56-year-old male patient complains of A. Lupus erythematosus pain in the left parotid-masticatory area and B. Chronic hyperplastic candidiasis progressing facial asymmetry first noticed C. Lupus vulgaris (Tuberculosis luposa) one month ago. The diagnosis is the cancer D. Lichen ruber planus of parotid gland - T2N2. What method of E. Secondary treatment is optimal in the given case? 57. A 7-year-old boy complains of increased body temperature, up to 38oC, headache, sore . Objectively: there are erosions on the slightly hyperemic mucosa of the , Крок 2 Stomatology (англомовний варiант, iноземнi студенти) 2015 рiк 8

A. Combined method coarse. The anamnesis states short-time pain B. Surgical extraction of neoplasm caused by chemical stimuli. What is the provi- C. Radiation therapy sional diagnosis? D. Chemotherapy E. Lymph nodes removal A. Acute initial caries B. Chronic initial caries 62. A 20-year-old male patient complains of C. Acute superficial caries sharp pain in the mouth, increase of body o D. Systemic hypoplasia of enamel temeperature up to 38, 5 C, headache and E. Dental fluorosis aching joints, general weakness. The disease onset was 3 days ago due to overexposure 66. A 47-year-old patient is undergoing dental to cold. Objectively: the lips red border has treatment of erosive/ulcerative form of li- hemorrhagic scabs, oral mucosa has large chen ruber planus. 1-month-long conservati- erosions and ulcers merging with each other ve therapy was uneffective. What physi- and covered with greyish-white coating on the otherapeutic method is the most effective as background of diffuse hyperemia. Conjuncti- a part of complex therapy? vitis is observed. The skin of the forearms has erythematous spots sized 1,5 cm in diameter, A. Cryodestruction with blisters in their center. What is the most B. Ultra-high-frequency treatment probable diagnosis? C. Electrophoresis D. Ultraviolet irradiation A. Stevens-Johnson syndrome E. D’arsonvalization B. Erythema multiforme C. Drug-induced stomatitis 67. A 21-year-old patient complains of a D. Lyell’s syndrome constant progressing throbbing pain in the 27 E. Behcet’s syndrome tooth. Objectively: a large carious cavity is fi- lled with softened dentin, the tooth cavity is 63. A 36-year-old female patient has complai- closed. Probing of the cavity floor is painless, nt of the 23rd tooth filling constantly falling percussion causes acute pain, there is grade out. Objectively: the 23rd tooth has carious II tooth mobility. Palpation of the mucous cavity of the V class according to the Black’s membrane in the projection of the root apex of classification of caries lesions. A dentist has the 27 tooth causes pain. Radiological changes made a decision to restore anatomical shape of are absent. What is the most likely diagnosis? the tooth with dental inlay. What is the speci- fic feature of the cavity preparation for dental A. Acute purulent periodontitis inlay in this case? B. Exacerbation of chronic periodontitis C. Acute diffuse pulpitis A. Designing oval-shaped cavity D. Acute serous periodontitis B. Extention of the cavity floor E. Acute purulent pulpitis C. Creation of retention points D. Creation of additional platform 68. A child is 13 years old. The third upper E. Making cavity outlet to the palatal surface tooth is fully cut, but situated slightly above the occlusion surface. What kind of abnormal 64. A 68-year-old patient, having suffered from placement is it? stroke, has time to time bouts of retching, which lead to unilateral diclocation of the A. Supraversion temporormandibular joint. Setting the joint B. Torsoversion was possible only with anaesthesia. Objecti- C. Vestibular vely: absent are the following teeth - from the D. Infraversion 21st to the 28th, 15th, 16th, 17th, 18th, 36th, E. Oral 37th, 44th, 46th, 47th. What kind of denture construction will be optimal for the duration 69. A 47-year-old patient complains of ulcer on of treatment? the back of the tongue. The anamnesis states that the ulcer had been preceded for a long A. Removable laminar denture with mouth term by protruding infiltrate that was gradually opening restrictor growing until the ulcer appeared. Objectively: B. Schroder appliance with sliding joint the back of the tongue has shallow oblong C. Petrosov appliance with restrictor ulcer surrounded with bumps with undermined D. Yadrova appliance edge. What is the most probable diagnosis? E. Khodorovych-Burgonska appliance with restrictor A. Tuberculous ulcer B. Primary syphilis 65. Parents of a 12-year-old child are concerned C. Tongue actinomycosis about the child having white spots on the D. Tongue abscess frontal teeth of the upper jaw; the spots E. Decubitus ulcer appeared half a year ago. Objectively: there are chalky spots detected in the cervical zone 70. A 16-year-old patient has made an appoi- of the 11th, 12th, 13th, 21st, 22nd, 23rd teeth ntment with a doctor due to the following vestibular surface. The enamel in those spots complaints: enlarged lymph nodes and fistula is dull; probing revealed it to be pliant and in the submandibular area; general weakness; Крок 2 Stomatology (англомовний варiант, iноземнi студенти) 2015 рiк 9 low grade fever. Objectively: the submandi- brown exudate without cholesterol crystals. bular lymph nodes are dense and slightly pai- What is the provisional diagnosis? nful, with clearly detectable margins. Caseous substance is produced from the fistulous tract. A. Osteoclastoma What is the most probable diagnosis? B. Mandubular hemangioma C. Adamantinoma A. Tuberculosis of lymph nodes D. Mandubular cyst B. Actinomycosis of lymph nodes E. Cholesteatoma C. Syphilitic lymphadenitis D. Chronic osteomyelitis 75. A 20-year-old female patient complains E. Subcutaneous granuloma of pain caused by eating sweet or sour food. Objectively: the vestibular surface of the 11th 71. A 42-year-old patient complains of pain tooth has a chalky spot, coarse when probed. in the right side of her head, restricted The centre of the spot has defect localised in movements of the lower jaw, clicking sound, the enamel. Fedorov-Volodkina index equals periodic spasms of chewing muscles. Objecti- 2,5 (Oral Hygiene Index). What is the most vely: the face is symmetric, mouth openi- probable diagnosis? ng is restricted. On palpation of the right temporo-mandibular joint (TMJ) there are A. Acute superficial caries crepitation and clicking accompanying mandi- B. Acute median caries ble movements. Examination of the oral cavity C. Enamel hypoplasia revealed also a Kennedy’s class II defect on the D. Initial caries right. What is the most likely diagnosis? E. Enamel necrosis A. Pain dysfunction of the right TMJ 76. A 50-year-old male patient complains B. Acute arthritis of the right TMJ of "small sore"on the sublingual mucosa. C. Sclerosing osteoarthritis of the right TMJ Pathomorphologic study of tissue sampli- D. Contracture of the right TMJ ng revealed the following: polymorphism E. Myositis ossificans of spiny epithelial cells has progressed into atypism; increased mitosis; giant and multi- 72. A 13-year-old child suffers from nucleate cells; acanthosis; hyperkeratosis and odontogenic osteomyelitis starting from the parakeratosis; basal membrane and basal layer 36th tooth and complicated by abscess of the are retained. What is the most probable di- . The 36th tooth is agnosis? to be extracted; it is necessary to open the pterygomandibular space. These operations A. Bowen’s disease require the following type of anaesthesia: B. Leukoplakia, erosive form C. Lichen ruber planus, hyperkeratotic form A. General anaesthesia D. Lupus erythematosus, erosive/ulcerative B. Веrcher-Dubov central anaesthesia form C. Infiltration anesthesia E. Decubitus ulcer D. Tuberal and palatinal anaesthesia E. Torusal anaesthesia 77. A 15-year-old adolescent complains of hi- gh body temperature up to 39, 5 − 40, 0oC, 73. Clinical stage of checking the clasp (bugel) vomiting, headache and sore throat, especially dental prosthesis framework as it should be when swallowing. Objectively: oral mucosa is placed in the oral cavity revealed the denture swollen, hyperemic. The patient is diagnosed arch to come off unevenly from the hard palate with acute catarrhal gingivitis. The tonsils are and alveolar process creating the gap up to 0,8 hypertrophic and covered with yellow-grey mm in width. What method can be applied to coating that does not spread from lymphoid remove this defect? tissue and is easily removed. Submandibular, cervical, and occipital lymph nodes are signi- A. To make a new framework for clasp dental ficantly enlarged since the very first day of prosthesis illness and are painful when palpated. The li- B. To make adjustments to the framework using ver and spleen are enlarged. What is the most crampon forceps probable diagnosis? C. To make adjustments to the framework using dental hammer and bench anvil A. Infectious mononucleosis D. To heat the metal using a gasoline blow torch B. Scarlatina and make adjustments by pressing to the model C. Diphtheria E. This kind of defect does not require D. Herpetic angina adjustments E. Measles 74. A 48-year-old man has an X-ray image of 78. A patient complains of loss of feeling in his lower jaw made during sanation of the oral the left side of his lower lip and chin, whi- cavity. Destruction of the bone tissue in the ch occurred after the complicated extraction gonial angle area is detected, the pathology is of the 37th tooth. Electric pulp test (EPT) not connected with tooth roots. The affected revealed decrease in electroexcitability of the area has clear margins. Puncture consists of left lower jaw teeth. What is the most probable Крок 2 Stomatology (англомовний варiант, iноземнi студенти) 2015 рiк 10 diagnosis? A. Lipoma B. Brancial cleft cyst A. Neuritis of the left inferior dental nerve C. Retention cyst of the left submandibular B. Neuralgia of the left inferior dental nerve C. Alveolitis of the 37th tooth socket D. Hemangioma D. Herpes Zoster n.Trigemini E. Chronic lymphadenitis E. Acute osteomyelitis of the body of mandible 84. A 52-year-old patient complains of pain 79. During the extraction of the 17 tooth the di- and a swelling in the right parotid region. stal buccal root broke off and remained in the These manifestations have been present for socket. Select a tool to extract the root: about 2 years. Over the last month the swelli- ng has grown bigger, pain has intensified. A. Bayonet forceps, straight elevator Objectively: the face is asymmetric due to the B. Straight forceps, straight elevator dense infiltrate in the right parotid region. The C. S-shaped forceps poorly circumscribed, painful formation infi- D. Beak-shaped forceps, angular elevator ltrates the surrounding tissues. At the right E. Angular elevators side of neck in front and behind the sternoclei- domastoid muscle there are enlarged, dense, 80. A partial removable denture is being made mobile lymph nodes. The right naso-buccal for a 50-year-old patient. What impression groove is flattened, the corner of mouth is trays are the more advisable? downturned. The mouth opens freely. The are A. Split stock tray pronounced symptoms of the right facial nerve B. Metal stock tray paresis. What disease can be suspected? C. Disposable stock tray A. Adenocarcinoma of the parotid salivary D. Perforated stock tray gland E. Customized stock tray B. Chronic parotitis 81. An obturator is being made using Ilyina- C. Actinomycosis of the parotid-masseter Markosyan technique for a 45-year-old-patient region to compensate for the defect of the hard and D. Chronic lymphadenitis soft palate. What type of connection between E. Pleomorphic adenoma of the parotid gland the fixing and obturating parts will be used? 85. A 48-year-old patient complains of the A. Button lower jaw teeth mobility. Van Thiel dental spli- B. Band nt is to be made for prosthodontic treatment. C. Clasp What construction elements are supposed to D. Joint fixitinplace? E. Spring A. Whole piece proximal grip clasps 82. A 10-year-old child has fallen and hit stairs B. Full metal crowns with his teeth. An appointment with a dentist C. Wire clasps was made only 2 days later. There are complai- D. Parapulpar posts ns of cold stimulus causing sharp pain. Objecti- E. Equator crowns vely: the 11th tooth has transversal defect 86. A 38-year-old patient has made an appoi- at 2/3 of the crown, the pulp is significantly ntment with a dentist to extract the 46th tooth exposed. Probing causes sharp pain. Percussi- in preparation for prosthetics. Objectively: on is painful, the tooth is mobile. The gums are the 46th tooth is destroyed by 2/3. What ki- slightly hyperemic. What method of treatment nd of anaesthesia is the most advisable for the should be used in this case? extraction of the 46th tooth? A. Vital extirpation A. Torusal B. Non-vital extirpation B. Веrcher-Dubov C. Biological method C. Mandibular D. Vital amputation D. Infiltration E. Non-vital amputation E. Mental 83. A 33-year-old patient complains of slowly 87. A 13-year-old boy complains of general growing tumor mass in the left submandi- weakness, high body temperature up to 39oC, bular area. Objectively: a tumor with soft lack of appetite, constant pain in the body of elastic consistency can be palpated in the left the lower jaw. Objectively: observed is signifi- submandibular area; the tumor changes its cant asymmetry of the face caused by soft ti- configuration when squeezed with fingers and ssues swelling in the left buccal and submandi- is painless. The skin covering the tumor has bular areas. Mouth opening is restricted. no changes in colour, movable. There are no Intraoral examination revealed the following: changes detected in the oral cavity. Salivary the 34th, 35th, 36th and 37th teeth are mobi- glands function normaly. Shrinking-inflation le; teeth percussion is painful. The crown of symptom is absent. What is the most probable the 36th tooth is completely destroyed. The diagnosis? mucosa of those teeth is hyperemic and painful when palpated. Muff-like enlargement of the Крок 2 Stomatology (англомовний варiант, iноземнi студенти) 2015 рiк 11 lower jaw alveolar process is detected. What is immobile teeth. The marginal periodontium the most probable diagnosis? in the area of the 14th-11th, 21st-23rd teeth has inflammation symptoms; gingival and peri- A. Acute mandibular odontogenic osteomyeli- odontal pockets are 4-5 mm deep. Whole piece tis metal and plastic denture construction is to be B. Acute mandibular hematogenous made for the patient. What are the specifics osteomyelitis of the construction edge placement relative to C. Acute mandibular odontogenic purulent the marginal periodontium in this case? periostitis D. Ewing’s sarcoma A. Circular garland covering necks of all E. Abscess of the right submandibular area abutment teeth B. Formation of gum-level circular ledge 88. A 25-year-old woman complains of pain C. Formation of subgingival circular ledge in the 15th tooth, which is caused by thermal D. Construction should be composed of stimuli and quickly ceases, when stimulation separate crowns stops. Objectively: visually detected is a defect E. Construction should have neither lining nor with smooth walls that join under an angle, circular ledge which is located in the cervical zone of the 15th tooth vestibular surface. Thermometry is pai- 92. Parents of an 8-year-old child with Down nful; percussion is painless. What is the most syndrome took the child to a doctor for oral probable diagnosis? cavity sanitation. After the examination entai- ling great difficulties the child was found to A. Cuneiform defect have four teeth decayed as a result of chronic B. Enamel erosion periodontitis. What kind of anesthesia should C. Enamel hypoplasia be chosen for surgical sanitation in one visit? D. Fluorosis E. Superficial caries A. Phlebonarcosis B. Conduction anesthesia 89. A 40-year-old female patient complains of C. Mask anesthesia short-term pain in the 34th tooth caused by D. Endotracheal anesthesia thermal stimuli. Objectively: the distal surface E. Endotracheal anesthesia through a of the 34th tooth has a carious cavity localised tracheostoma in the circumpulpar dentin. The walls and floor of the cavity are softened, faintly pigmented; 93. During examination of the 11-year-old chi- probing of the floor is slightly painful. Cold sti- ld’s oral cavity the 23rd tooth vestibular posi- mulus causes pain that ceases when stimulati- tion was detected. Correlation of the 16th and on stops. What is the most probable diagnosis? 46th is Angle class 1, and 26th and 36th is Angle class 2. The width of the 23rd crown A. Acute deep caries is 8 mm. The dentition lacks 4 mm to place the B. Acute median caries 23rd properly. Front teeth occlusion is normal. C. Chronic deep caries Choose the optimal treatment approach. D. Chronic median caries E. Chronic fibrous pulpitis A. Move the upper lateral teeth on the right side in distal direction, than move the canine 90. A 24-year-old patient complains of new into the correct place growth in the right lateral surface of the neck. B. Extract the canine; move the 24th and 25th The new growth was first noticed 4 months teeth to replace the 23rd ago, no appointment with doctor was made. C. Widen the upper and lower dentitions, and Objectively: the face is asymmetrical due to the move the canine into the correct place new growth in the upper third part of the right D. Widen the upper dentition, and move the lateral surface of the neck. Palpation allows to canine into the correct place detect the painless rounded new growth sized E. Extract the first premolar, and move the 3х3,5 cm; its consistency is dence and elastic; canine into the correct place the skin abowe the growth has no changes of colour and can be folded. The new growth is 94. A 47-year-old patient complains of a burni- not melded with its surrounding tissues and is ng sensation and pain in the mouth. Objecti- situated along the front edge of the sternoclei- vely: on the mucous membrane of cheeks along domastoid muscle. Puncture consists of pale the line of teeth contact and in the corners of yellow viscous fluid. What is the most probable mouth there are multiple polygonal bright red diagnosis? erosions 1,0-1,5 cm in diameter located on the hyperkeratinized plaque and opaque whitish A. Brancial cleft cyst mucosa. Cytological analysis revealed kerati- B. Phlegmon of neck lateral surface nizing epithelial cells. What is the most likely C. Chronic lymphadenitis of neck lateral diagnosis? surface D. Lipoma of neck lateral surface E. Acute serous lymphadenitis of neck lateral surface 91. A female patient is 51-year-old, with Крок 2 Stomatology (англомовний варiант, iноземнi студенти) 2015 рiк 12

A. Leukoplakia, erosive form A. Local precancerous hyperkeratosis of the B. , erosive form lower lip C. Erythema multiforme B. Cheilitis abrasiva praecancerosa Manganotti D. Secondary syphilis C. Verrucous precancer of the lower lip red E. Lupus erythematosus, erosive form border D. Bowen’s disease 95. A 24-year-old female patient complains E. Erythroplasia of Queyrat of the 15th tooth being lost. The defect is of aesthetic nature. Objectively: the 14th and 99. A 23-year-old male patient complains of 16th teeth are intact, stable, clinical crowns gum bleeding when he brushes his teeth or eats are tall with pronounced equators and normal tough food. Objectively: the gums of the front orthognathic overlapping; X-ray image shows lower jaw are hyperemic, swollen and bleed no pathologies in the periapical tissues. What when palpated. The oral and gingival mucosa kind of denture should be recommended for in other areas are not affected. The occlusi- the patient? on is deep. The teeth are firm, except for the 41st and 31st (class 1 mobility). X-ray imagi- A. Adhesive dental bridge ng shows resorption of the alveolar septum in B. Metal ceramic dental bridge the area of the 41st, 42nd, 32nd and 31st teeth C. Metal plastic dental bridge up to 1/3 of the root length. What is the most D. Plastic dental bridge probable diagnosis? E. Cantilever bridge A. Localised periodontitis 96. A12-year-old child has a carious cavity B. Generalised periodontitis, initial stage on the surface of the 12th tooth. The cavi- C. Generalised periodontitis, stage I ty is localised in the mantle dentin; its floor D. Catarrhal gingivitis and walls are covered with pale softened E. Parodontosis, stage I dentin. Probing the enamel-dentin border is painful. Thermal stimuli cause short-term pai- 100. A 78-year-old patient complains of diffi- nful reaction. Vertical percussion is painless, culties when taking food and deformity of the horisontal percussion is slightly painful. Gingi- dentition. The anamnesis states the lower jaw val papilla between the 12th and 13th teeth fracture 2,5 month ago. Objectively: there is a is swollen, hyperemic, bleeds when touched. deformity of the lower jaw front area, which What is the most probable diagnosis? equals 19 mm (artificial joint). Both fragments of the lower jaw have 3 stable teeth each. What A. Acute median caries, gingival papillitis prosthesis construction is the most advisable in B. Acute deep caries, gingival papillitis this case? C. Acute partial pulpitis, gingival papillitis D. Acute serous periodontitis A. Oxman joint fixed denture E. Exacerbation of chronic periodontitis B. Oxman joint denture with one pivot-point C. Jointless removable denture 97. A 27-year-old female patient has her 26th D. Kurlandsky’s denture with shock absorbing tooth cavity accidentially breached along the ball-clasp mesial buccal line angle during treatment of E. Oxman joint denture with two pivot-points acute deep caries of the 26th. Choose the opti- mal method of treatment. 101. A 3,5-year-old child has symmetrical face, the middle part is predominant in proporti- A. Biological method ons, swallowing is infantile, breathing is nasal. B. Non-vital extirpation In the oral cavity the dentition corresponds C. Non-vital amputation with the age norms, the sagittal fissure is 3 D. Vital amputation mm, every tooth in the lateral part has its E. Vital extirpation anatagonist, the lower teeth touch the hard 98. A 54-year-old patient complaining of palate. Miogymnastics with Dassa orbicularis tumor-like growth on the red border of the oris activator is recommended. What function left lower lip has made an appointment with is normalised by this apparatus in the given a dentist. Examination revealed the affected case? area to be sized 1х1,5 cm, and have irregular A. Lips closure shape with clear margins. Focus of the affected B. Breathing area does not protrude from the surrounding C. Chewing red border; the affected area is greyish-white D. Swallowing in colour and covered in thin closely placed E. Speech scales. What is the most probable diagnosis? 102. A patient consulted a dentist about pain in the 47 tooth. X-ray examination revealed fragments of endodontic instruments in the canals of the medial and distal root. In the periapical zone there are well-defined areas of bone destruction. Select the most appropriate method of treatment: Крок 2 Stomatology (англомовний варiант, iноземнi студенти) 2015 рiк 13

A. Tooth extraction A. Korkhaus appliance B. Radectomy B. Vasylenko appliance C. Tooth replantation C. Simple cotton ligature D. Tooth hemisection D. Kalvelis appliance E. Conservative treatment E. Begg appliance 103. After unilateral resection of the upper jaw 108. An injured patient complains of limited a 52-year-old patient received the immediate- opening of the mouth, nose bleeding, skin insertion denture. What is the term of its use? numbness in the infraorbital and lower eyelid region. Objectively: there is face deformation A. 1-3 months due to the depression of soft tissues in the left B. 1-2 months cheekbone region, step deformity in the mi- C. 7-12 months ddle part of the inferior margin of the left orbit D. 2-3 years and in the area of the zygomatic alveolar crest. E. 4-5 years What is the most likely diagnosis? 104. A 57-year-old female patient complains of A. Zygomatic bone fracture with displacement burning pain, dry mouth, feeling of foreign of bone fragments body on the tongue back and lateral surfaces, B. Fracture of the right zygomatic bone without which disappears in the process of eating. displacement of fragments The first case of such symptoms was noted C. Le Fort I fracture of maxilla a year ago after psychic trauma. The patient D. Le Fort II fracture of maxilla suffers from pancreatic diabetes and sleep di- E. Fracture of the malar arch sturbance. Examination revealed the followi- ng: the tongue mucosa is without significant 109. A patient has sustained blunt object changes, moderately moistened. What is the trauma to the face central area. Objectively: most probable diagnosis? the nasal bones are mobile, there is signifi- cant swelling of the left temporal region soft A. Glossodynia tissues, uneven "stair-like"shape of bones is B. Candidal observed in the lower edges of both eyesockets C. Glossitis areata exfoliativa and maxillo-zygomatic sutures, bleeding, open D. Hunter-Moeller’s glossitis bite. The central part of the face is elongated. E. Glossitis with fissured tongue What is the most probable diagnosis? 105. A patient presented to a dental clinic for A. Le Fort fracture of maxilla, type II complex dental care. Objectively: the 37 tooth B. Le Fort fracture of maxilla, type I exhibits a deep carious cavity communicati- C. Left temporal bone fracture ng with the tooth cavity. There is no response D. Nasal fracture to stimuli. Radiography reveals widening and E. Le Fort fracture, type III deformation of the periodontal ligament space in the apical region. What is the most likely 110. A 44-year-old female patient has complai- diagnosis? nts of the face swelling at the right lower jaw area and teeth mobility. Objectively: A. Chronic fibrous periodontitis soft tissues are without changes, the regional B. Chronic granulating periodontitis lymph nodes cannot be palpated. The alveolar C. Chronic granulomatous periodontitis process and the body of the lower jaw near the D. Chronic fibrous pulpitis 46th, 47th and 48th teeth are thickened, pai- E. Chronic gangrenous pulpitis nless when palpated, have bulges. The teeth in the thickened area are mobile. Puncture consi- 106. A patient with post-resection upper jaw sts of brown fluid without cholesterol crystals. defect that invades the nasal cavity has made What is the provisional diagnosis? an appointment with a prosthodontic clinic. What denture is recommended in the given A. Osteoclastoma case? B. Adamantinoma C. Osteoma A. Replacement denture with obturating D. Follicular cyst element E. Odontoma B. Floating obturator C. Mouthguard 111. A 49-year-old patient consulted a dental D. Forming denture surgeon about the oral cavity sanitation. He E. Replacement denture has an indication for the extraction of the 16 tooth. History: the tooth hasn’t been treated 107. An 18-year-old patient with complaint of before, it has decayed within the last 4 years. large diastem has made an appointment with Objectively: the 16 tooth’s crown is decayed by prosthodontics specialist. Objectively: there is over 2/3, the mucosa exhibits no pathological full lateral displacement of central incisors due changes. Which tool is required for the tooth to absence of the 12th and 22nd teeth. What extraction? instrument is the most advisable for moving the central incisors closer together? Крок 2 Stomatology (англомовний варiант, iноземнi студенти) 2015 рiк 14

A. Bayonet root forceps is semi-impacted. Palpation of the internal B. S-shaped forceps (right) surface of the mandibular angle is painful. C. Straight elevator What is the most likely diagnosis? D. S-shaped closed-beak forceps E. Crown bayonet forceps A. Phlegmon of pterygomandibular space B. Acute tonsillitis 112. A 39-year-old patient, who works in a C. Phlegmon of the infratemporal region printing house, complains of burning pain, D. Acute of the 38 tooth region metallic taste in the mouth and bad breath. E. Phlegmon of peripharyngeal space Objectively: the skin is sallow in colour. The oral mucosa has black-and-blue spots. The 117. A 35-year-old patient complains of a gums are hyperemic and swollen. The edge of neoplasm on the tip of his tongue which he the gums has black-and-blue band. The palate injures with his teeth. The neoplasm someti- is of yellow shade. Intoxication symptoms mes increases, and sometimes decreases in si- are observed. What is the most probable di- ze. Objectively: on the tip of tongue there is a agnosis? roundish well-defined neoplasm 0,5 cm in di- ameter with broad base. The neoplasm is the A. Lead stomatitis same colour as the mucosa of tongue. What is B. Mercurial stomatitis the most likely diagnosis? C. Bismuthic stomatitis D. Melanosis A. Papilloma of tongue E. Addison’s disease B. Abscess of tongue C. Lipoma of tongue 113. A 16-year-old female patient complains of D. Hemangiofibroma of tongue a deformation and restricted mouth opening E. Fibroma of tongue since birth. Objectively: the face is symmetri- cal, disproportionate, there is a bird-like face 118. A 37-year-old patient complains of bleedi- symptom. The mouth opens up to 1 cm. The ng gums, bad breath, loose teeth, difficulti- patient has an occlusal abnormality, namely es with chewing food. Objectively: gums are deep incisal overbite. What pathology is found bluish-red in colour; periodontal pockets are in this patient? 6 mm deep and contain serous substance; class II teeth mobility; moderate deposits of A. Micrognathia with congenital ankylosis subgingival and supragingival dental calculus; B. Micrognathia with acquired ankylosis Oral Hygiene Index equals 3 points; there is C. Maxillary protraction traumatical occlusuion along all the length of D. Prognathism the dental arches. What is the provisional di- E. Mandibular protraction agnosis? 114. To perform a differentiated diagnostics of A. Generalized periodontitis, stage II, chronic the lower jaw displacement a patient was asked B. Generalized periodontitis, stage I, acute to open his mouth as wide as possible, and then C. Hypertrophic gingivitis the lateral displacement of the lower jaw and D. Histiocytosis X face asymmetry were measured. What clinical E. Generalized periodontitis, stage II, acute test is it? 119. A 6,5-year-old child has closed non- A. Ilyina-Markosyan clinical functional test 3 pigmented fissures in the first permanent B. Ilyina-Markosyan clinical functional test 1 molar, which have been revealed during C. Ilyina-Markosyan clinical functional test 2 preventive examination. Enamel transparency D. Ilyina-Markosyan clinical functional test 4 is retained, its probing does not reveal any E. Eschler-Bittner clinical diagnostic test coarseness. Choose the optimal method of treatment in this case. 115. What measurements are necessary to determine the width of dental arch accordi- A. Non-invasive sealing ng to the Pont’s method of analysis? B. Invasive sealing C. Preventive filling A. Crown width of four upper incisors D. ART technique B. Crown width of upper central incisors E. Regular medical check-up C. Crown width of six upper front teeth D. Crown width of upper central incisors and 120. A 3-year-old child suffers from extreme the first premolars gingivitis with slight teeth mobility; teeth are E. Size of dentition frontal segment intact. The skin is dry, the hair is brittle, the skin of the palms and soles of the feet is rough 116. A patient complains of limited mouth and covered in small cracks. What is the provi- opening, pain during swallowing, fever up to sional diagnosis? 38, 5oC, weakness, indisposition. Objectively: the mouth opens up to 1 cm. After Berchet anesthesia examination of the oral cavity revealed edema, hyperemia, tenderness of the pterygomandibular fold. Lateral and posteri- or pharyngeal walls are intact. The 38 tooth Крок 2 Stomatology (англомовний варiант, iноземнi студенти) 2015 рiк 15

A. Papillon-Lefevre syndrome A. Loosely fill the socket with iodoform tampon B. Niemann-Pick disease B. Fill the socket with hemostatic sponge C. Hand-Schuller-Christian disease C. Lavage the socket with microcide solution D. Eosinophilic granuloma (Taratynov’s di- D. Lavage the socket with 0,1% chlorhexidine sease) solution E. Letterer-Siwe disease E. Fill the socket with antibiotic dusting powder 121. Parents of an 8-year-old child have made 125. A 24-year-old female patient consulted and appointment with an orthodontist. There a dentist about pain in the 26 tooth. After are complaints of their child having traumas a physical examination the patient was di- of oral mucosa. Objectively: decreased hei- agnosed with exacerbation of chronic peri- ght of the face lower part, everted lower lip, odontitis of the 26 tooth. During the tooth deep labiomental furrow, occlusion. The extraction the coronal part of the tooth was upper incisors fully cover the lower ones; cutti- accidentally broken. Further manipulations wi- ng surface of the lower incisors make contact th bayonet forceps failed to extract the tooth with the anterior third of the palate. Mesiodi- roots. What actions are to be taken for the stal ratio of the canines and the first permanent successful root extraction? molars is normal. Grouping of the upper and lower front teeth is dissimilar. Make the di- A. To disjoin the roots by using a drill and agnosis according to the Kalvelis classification. fissure burr B. To send the patient to the hospital A. Deep traumatic overbite C. To use Lecluse elevator B. Deep incisor overbite D. To complete the tooth extraction during the C. Deep neutral occlusion next visit D. Deep prognatic (roof-shaped) occlusion E. To use angular elevator E. - 126. Preventive examination of a 5-year-old 122. A 64-year-old patient with edentulous child revealed a habit of lower lip biting. What jaws has a minor uniform atrophy of maxilla malocclusion may develop if the child keeps and a pronounced atrophy of lateral parts of this habit? mandible. The mucosa in these parts makes longitudinal folds, the alveolar crest is mobile A. Anterior bite in the anterior part. The condition of mandi- B. Prognathic bite ble should be taken into consideration at the C. Open bite following stage of fabrication of a complete D. Deep overbite removable denture: E. Cross-bite A. Taking differentiated functional impression 127. Parents of an 8-year-old girl are concerned B. Taking anatomical impression that she is chewing food too slowly. Objecti- C. Taking compression functional impression vely: it is a period of transitional dentition. D. Taking decompression functional impression The first permanent molars has neutral relati- E. Determining the central occlusion onship; sagittal fissure is 2 mm wide. The upper front teeth cover the lower ones by 2/3. Name 123. A 46-year-old patient complains of masti- the pathology. cation disruption caused by the lack of the 34th, 35th and 36th teeth. The antecedent A. Deepening of incisor overbite anamnesis is as follows: the teeth were B. Widening of the sagittal fissure extracted 3 months ago due to complication C. Tooth-alveolar lengthening of the front teeth of cariosity. The patient anamnesis: the history D. Tooth-alveolarshortening of the lateral teeth of tonsillitis, rheumatoid arthritis and Botki- E. Decrease of the lower face height n’s disease. After the appointment with this 128. When checking construction of the patient instruments should be sterilized in the soldered dental bridge with the 35th and 38th following way: abutment teeth the following was detected: A. Specialized procedure pores in the place of soldering together B. Dry-heat sterilizer abutment crowns with the intermediate part; C. Processing with lysol masticatory tubercules are sharply defined; D. Processing with 0,1% chloramine solution there is early contact with tooth-antagonists; E. General procedure the intermediate part makes tight contact with the alveolar process mucosa. How can those 124. A 49-year-old patient has a tooth flaws be corrected? removed from the left upper jaw under plexus anaesthesia with articaine-forte (Articaine + Epinephrine). After the operation the tooth socket did not fill with blood clot. How to prevent alveolitis in this case? Крок 2 Stomatology (англомовний варiант, iноземнi студенти) 2015 рiк 16

A. Dental bridge shoold be remade ent? B. Intermediate part should be corrected C. Masticatory surface should be corrected, and A. Partial removable laminar metal-based soldered places - filed denture D. Masticatory surface should be corrected, and B. Dental bridge the height of the flushing part is to be increased C. Partial removable laminar plastic denture up to 2 mm with clasp-retainers E. Tooth-antagonists sholud be filed down D. Partial removable laminar denture with supporting-retaining clasps 129. A 14-year-old girl complains of indisti- E. Clasp (bugel) dental prosthesis nct pronunciation that developed at the age of 14 after the acute respiratory viral disease. 133. A 22-year-old patient presented to a Examination revealed normal face and normal prosthetic dentistry clinic because of missing teeth alignment, occlusal disharmony was not of the 21 tooth, the 11 and 22 teeth are intact. found. Palpation didn’t reveal cleft palate. The 21 tooth was extracted 2 months ago. What Uvula doesn’t move during pronunciation of construction is most suitable in this case? sounds, its palpation does not cause gag reflex. What is the reason for indistinct pronunciation A. Two-stage implantation of sounds? B. One-stage implantation with simultaneous fabrication of porcelain-fused-to-metal crown A. Paresis of the soft palate and uvula muscles C. Partial removable denture B. Adenoid vegetations D. Swaged and soldered denture supported by C. Palatal slit the 11 and 22 teeth D. Hypertrophy of lingual tonsil E. Metal-plastic denture supported by the 11 E. Deformation of the bite and 22 teeth 130. A 53-year-old male patient has made 134. A 48-year-old patient has undergone uni- a repeat appointment after 3 days upon the lateral resection of the upper jaw. He needs extraction of the 36th tooth. Spot X-ray of the a resection prosthesis. Objectively: the 21, 22, 36th tooth made from within the oral cavity 23, 24, 25, 26 teeth are stable, hard palate is revealed the remaining mesial root. What tools broad, flat. What connection of clasps with the should be used to extract this root? base will reduce the instability of the resection prosthesis? A. Inward bent elevator and upper beak-shaped forceps without spike A. Articular B. Outward bent elevator and upper beak- B. Stable shaped forceps without spike C. Elastic C. Inward bent elevator and lower beak-shaped D. Fixed forceps without spike E. Does not matter D. Outward bent elevator and lower beak- shaped forceps without spike 135. A 15-year-old girl complaining of E. Inward and outward bent elevators and moderate swelling of the left parotic- upper beak-shaped forceps without spike masticatory area has made an appointment wi- th a dental surgeon. Palpation revealed the left 131. A 35-year-old female patient seeks parotid gland to be bulgy, dense, and painless. prosthetic dentistry. Objectively: the 18, 14, 13, Its duct secretes turbid saliva. The duct orifice 12, 11, 21, 22, 23, 24, 28 teeth need restoration. is dilated, the surrounding mucosa is cyanotic, The crowns confining the defect are high and pastose, with teeth imprints. What disease is it? stable. To restore the integrity of the dentition it is planned to fabricate a cast clasp denture. A. Chronic parenchymatous sialoadenitis What kind of mechanical fixator should be B. Chronic interstitial sialoadenitis used? C. Calculous () D. Mixed tumor A. Bar fixation E. Cyst B. Supporting-retaining clasps C. Retaining clasps 136. The department of maxillofacial surgery D. Telescopic fixation admitted a patient who needs repair of a post- E. Dentoalveolar clasps traumatic defect of nose wing up to 3,0 cm in diameter. The trauma occured six months ago. 132. A 42-year-old female patient has What kind of grafting is indicated in this clini- made an appointment with a prosthodontic cal situation? office to make a denture. Objecti- vely: dental formula is as follows: 18....131211 212223....28 48 47 46 45 44 43 42 41 31 32 33 34 35 36 37 . The patient has deep occlusion; clinical crowns are low; equator is not pronounced. The pati- ent suffers from epileptic seizures. What kind of denture should be prescribed for this pati- Крок 2 Stomatology (англомовний варiант, iноземнi студенти) 2015 рiк 17

A. Grafting with chondrocutaneous flap of the A. Mandibular anaesthesia with 3% auricle Scandonest solution B. Grafting with local tissues of nasolabial or B. Intravenous anaesthesia with thiopental cheek regions sodium solution C. Grafting with pedicle flap of frontal and C. Infiltration anesthesia with 2% lidocaine buccal regions solution D. Grafting with tubed pedicle (Filatov’s) flap D. Mandibular anaesthesia with Ultracain DS E. Free grafting with dermal flap forte (epinephrine concentration 1:100 000) E. - 137. A 25-year-old female patient consulted a dentist about acute pain in the upper jaw on 141. During the surgical removal of a retention the left. The pain occurs during eating. Objecti- cyst of the lower lip a 14-year-old boy complai- vely: on the distal approximal surface of the ned of sudden weakness, dizziness, nausea. 26 tooth there is a cavity filled with light soft Objectively: the skin is covered with cold dentin. Probing causes a slight pain along the sweat. Respiration is frequent, pulse is weak, dentin-enamel junction, percussion is painless. AP is decreased (90/60 mm Hg), the hands are Cold water causes quickly abating pain. What cold. What is the most likely diagnosis? is the most likely diagnosis? A. Syncope A. Acute median caries B. Toxic shock B. Chronic median caries C. Traumatic shock C. Acute deep caries D. Anaphylactic shock D. Chronic fibrous pulpitis E. Collapse E. Chronic deep caries 142. A 10,5-year-old child complains of painful 138. A 40-year-old female patient has made an rash on his lips. Objectively: the red border appointment with a dentist. She complains of of the lips is swollen, hyperemic, covered in pain in her front teeth of the upper and lower fissures and numerous scabs of dried blood. jaws, which is caused by thermal, chemical and The skin of the upper lip has small blisters mechanical stimuli. Temporomandibular joint containing serous substance, which merge wi- is also slightly painful. Objectively: the pati- th each other in some places. Maceration and ent has deep occlusion; the enamel of lingual weeping skin also can be observed, especially surface of the front upper teeth and vestbular in the corners of the mouth. What is the most surface of the front lower teeth is rubbed off probable diagnosis? exposing the glossy pigmented dentin. What is the most probable diagnosis? A. Exematous cheilitis B. Meteorological cheilitis A. Pathologic brygmus C. Atopic cheilitis B. Chemical necrosis of the tooth D. C. Physiological brygmus E. Exfoliative cheilitis D. Hard tissues erosion E. Stainton-Capdepont syndrome 143. An 11-year-old child complains of dryness and tightness of the lips, which the child is li- 139. Whan conduction anaesthesia had been cking constantly.Objectively: red border of the administerd by a dentist, in 2 seconds a lips is dry, infiltrated, hyperemic and covered patient developed the following symptoms: in numerous thin scales. The skin of the lips is loss of consciousness, seizures, dilated pupils, undamaged. What is the provisional diagnosis? absent pupillary response to light, low arteri- al pressure, laboured breathing, muffled heart A. Meteorological cheilitis sounds. What is the most probable diagnosis? B. Atopic cheilitis C. Cheilitis exfoliativa A. Anaphylactic shock D. Allergic contact cheilitis B. Syncope E. Exematous cheilitis C. Collapse D. Cardiac infarction 144. Parents of a 7,5-year old child brought him E. Stroke to a dentist for oral cavity sanitation. Objecti- vely: DEF (for primary teeth) + DMF (for 140. A 13-year-old boy needs his 46th tooth permanent teeth) index = 4, Green-Vermillion extracted due to chronic granulomatous peri- index = 2,5. Fissures of the first permanent odontitis. He has pancreatic diabetes as a molars are open, intact, non-pigmented. What concomitant disease. Choose the most advi- method of primary prevention of dental caries sable anaesthetic and its way of administration. may be appropriate in this case? A. Non-invasive hermetization B. Invasive hermetization C. Application of fluoride varnish D. Application of antibacterial varnish E. Application of calcium gels 145. Parents of a 5-year-old child has made Крок 2 Stomatology (англомовний варiант, iноземнi студенти) 2015 рiк 18 an appointment with a dentist for preventi- ve examination of their child. Objectively: A. Osteosynthesis the DMF index (Decayed, Missing, Filled)=5, B. One arch smooth dental braces the gingival mucosa is pale pink in colour, C. Full dental splint Fedorov-Volodkina index is 2,5. For oral hygi- D. Weber splint ene the child should use a toothbrush with the E. Vankevytch splint following kind of bristle: 150. A 37-year-old patient had an appointment A. Soft with his orthopaedist about dental prosthetics. B. Medium During the preparation of hard dental tissues C. Hard for a fixed denture he was given conducti- D. Extra-soft on anesthesia. During the injection the pati- E. Extra-hard ent developed cold sweat, sudden pallor and cyanosis, thready pulse. The patient stayed 146. A 28-year-old patient complains of experi- conscious, but became relaxed and lethargic. encing gum discomfort, gingival haemorrhages What is the provisional diagnosis? and pain, especially during eating, for a week. Objectively: the gums are swollen and A. Collapse markedly hyperemic, especially within the 43, B. Anaphylactic shock 42, 41, 31, 32, 33 teeth, soft plaque and tartar C. Intoxication are present, is up to 2 mm deep. D. Syncope What is the most likely diagnosis? E. Hypertensive crisis 151. A 48-year-old patient is in grave conditi- A. Acute deep gingivitis o B. Chronic catarrhal gingivitis on: body temperature is of 39, 8 C, the face C. Hypertrophic gingivitis is asymmetric due to the infiltration of the D. Initial stage of chronic generalized peri- submental and both submandibular triangles. odontitis Speech is difficult as the tongue is displaced E. Exacerbation of initial generalized peri- upwards. Swallowing is painful. Sublingual pli- odontitis cae are markedly hyperemic and edematous. What is the most likely diagnosis? 147. An 18-year-old patient complains of chalky spot in the 23rd tooth, which is slightly A. Phlegmon of mouth floor painful when cold stimulus is applied. Objecti- B. Abscess of tongue root vely: the vestibular surface near dental cervix C. Phlegmon of submental triangle of the 23rd tooth has a white spot 2-3 mm in D. Phlegmon of alveololingual groove size. Probing is painless and reveals coarseness E. Phlegmon of submandibular triangle of the surface. Thermal stimulus causes slightly painful feeling. What is the most probable di- 152. A 32-year-old patient is hospitalised in agnosis? an oral surgery department with perforating wound of the cheek. The size of the wound A. Acute initial caries opening is 1,5х4,0 mm. What kind of restorati- B. Chronic initial caries ve surgical operation is the most advisable in C. Fluorosis this case? D. Local hypoplasia E. Enamel necrosis A. Filatov-Gillies tube graft B. Double-end flap 148. A 7-year-old child is diagnosed with C. Rotation flap complete dislocation of the 11th tooth caused D. Rauer’s double shoulder- flap by a trauma that happened 24 hours ago. E. Opposite triangular flaps (Z-plasty) The tooth was brought along. Objectively: the socket edges are slightly hyperemic, bone walls 153. A 60-year-old patient has been undergoi- are intact; the tooth socket is filled with blood ng the procedure of checking the complete clot. What approach is the most advisable for removable dentures construction and fixing the therapy? teeth on wax bases. The following flaws have been detected: fissure between the teeth on A. Reimplantation of the 11th tooth preceded the frontal area and tubercule contact in the by the canal filling lateral area. What mistake had been made? B. Reimplantation of the 11th tooth followed by the canal filling A. Anterior occlusion was determined instead C. Extraction of the 11th tooth of central one D. Reposition and fixation of the 11th tooth B. Posterior occlusion was determined instead E. Socket closure with catgut of central one C. Lateral occlusion was determined instead of 149. A 56-year-old patient suffering from central one exacerbation of schizophrenia has been hospi- D. Models were plastered in a wrong way in an talised in an oral in-patient department with a occluder diagnosis of the lower jaw displaced fracture in E. Swabs were crushed, when central occlusion the area of the 34th-35th teeth. What method was being determined of treatment should be prescribed? Крок 2 Stomatology (англомовний варiант, iноземнi студенти) 2015 рiк 19

154. A 20-year-old patient in the 8th month the following symptoms: tinnitus, weakness, of pregnancy complains about an aesthetic paleness of skin. Vertigo was diagnosed. What defect, speech disturbance due to the mi- first aid approach is the most advisable? ssing upper front tooth. Objectively: the 12 tooth is missing. The 11, 13 teeth are intact, A. Put the patient in the Trendelenburg positi- stable, of anatomical shape. The patient has on orthognathic bite. What prosthesic design B. Intramuscular injection of 50% analgin should be preferred? (mеtamisole sodium) solution C. Intravenous injection of 0,1% epinephrine A. Immediate prosthesis D. Subcutaneous injection of cordiamin B. Plastic bridge E. Intravenous injection of prednisolone C. Porcelain-fused-to-metal bridge D. Plastic-to-metal bridge 159. A 15-year-old adolescent complains of a E. Implant with a pocelain crown two-day long sharp pulsing pain in the tooth, which intensifies when biting or touching 155. A 30-year-old patient complains of pain the tooth with the tongue. Objectively: the and swelling in the area of the left parotid sali- 26th tooth contains composite material filli- vary gland, which occurred 7 days after he had ng. Vertical and horisontal percussion of the undergone abdominal cavity surgery. Objecti- tooth are painful, the tooth is slightly mobile vely: body temperature equals 39oC, restricted in vestibular-oral projection. Gingival mucosa mouth opening; dry mouth; when the gland of the affected area is hyperemic, swollen, is massaged, there is purulent exudate being sharply painful when palpated. X-ray imaging secreted from its duct. The patient can be di- shows no changes. What is the most probable agnosed with the following disease: diagnosis? A. Acute non-epidemic parotitis A. Acute purulent periodontitis B. Acute epidemic parotitis B. Acute purulent pulpitis C. Phlegmon of C. Acute serous periodontitis D. Parenchymatous parotitis D. Acute serous pulpitis E. Phlegmon of parotid-masticatory region E. Exacerbation of chronic periodontitis 156. A 16-year-old girl complains of cosmetic 160. A 5-year-old child has a cavity localised in defect of the front teeth - there are dark the deep dentin on the masticatory surface of spots and hard tissues defects. The spots were the 75th tooth. The cavity is filled with softened detected in the process of teething, the defects pigmented dentin and food remains. The chi- developed later. The following diagnosis was ld complains of dull pain when taking food or made: erosive form of fluorosis of the 16th, cold liquids. Probing of the cavity floor is sli- 11th, 12th, 22nd, 26th, 31st, 32nd, 36th, 41st, ghtly painful; when the dentin is removed with 42nd, 46th teeth. Choose the method of excavator, pink pulp is visible through the cavi- treatment. ty floor. What is the most probable diagnosis? A. Restoration treatment A. Chronic fibrous pulpitis B. Remineralization treatment B. Chronic deep caries C. Prosthetics C. Acute deep caries D. Surgical treatment D. Acute diffuse pulpitis E. Regular medical check-up E. Chronic gangrenous pulpitis 157. An 8-year-old boy has thickening of the 161. A 9-year-old child complains of dull pain lower jaw body in the area of the 83rd, 84th in a tooth, which increases when the tooth is and 85th teeth. In the given area a rounded pressed. One month ago the tooth was treated painless tissue protrusion can be detected; the for pulpitis. Objectively: the 36th tooth is fi- Dupuytren’s symptom is positive. Deciduous lled; percussion is sharply painful; mucosa is teeth are intact. X-ray image of the lower jaw hyperemic, swollen. X-ray imaging does not made in lateral projection shows the focus of show any changes. What is the most probable bone destruction with clear margins in the area diagnosis? of the 83rd, 84th and 85th teeth. The 44th tooth follicle is shifted down in distal direction, the A. Acute serous periodontitis crown is projected on the destruction focus. B. Exacerbation of chronic pulpitis What is the clinical diagnosis? C. Acute diffuse pulpitis D. Acute suppurative pulpitis A. Follicular cyst of mandibula from the 44th E. Exacerbation of chronic periodontitis tooth B. Fibrous dysplasia of mandible 162. A 50-year-old female patient complains C. Mandibular ameloblastoma of pain in the upper left jaw, high fever up D. Radicular cyst of mandibula from the 84th to 38, 5oC, worsening of her general conditi- tooth on. The disease onset was 3 days ago. Objecti- E. Mandibular osteoclastoma vely: the face is asymmetrical due to swelling of the right cheek soft tissues, palpation is pai- 158. In the process of the 26th tooth extracti- nful, the skin is hyperemic. The crown of the on a 34-year-old patient suddenly developed 16th tooth is destroyed completely; percussion Крок 2 Stomatology (англомовний варiант, iноземнi студенти) 2015 рiк 20 sensitivity is positive. There is hyperemia and A. Oral rinsing with sodium bicarbonate soluti- edema of the alveolar process soft tissues in the on, chlorhexidine solution area of the 15th, 16th and 17th teeth vestibular B. Teeth cleaning with chewing gum surface. What is the most probable diagnosis? C. Teeth flossing D. Teethbrushing using soft-bristled toothbrush A. Acute odontogenic maxillary periostitis in and anti-inflammatory toothpaste the area of the 16th tooth E. Teeth cleaning with interdental stimulators B. Exacerbation of chronic periodontitis of the and interdental brushes 16th tooth C. Acute odontogenic maxillary osteomyelitis 167. A 42-year-old female patient presented to starting from the 16th tooth a dentist for oral sanitation. She has a history D. Acute odontogenic right-side maxillary of tonsillitis, rheumatoid arthritis, Botkin’s di- sinusitis sease. What are the conditions of instrument E. Chronic odontogenic maxillary osteomyelitis treatment after the patient’s visit? 163. A 60-year-old patient complains of a A. Under the special scheme growth in the mental region, which is painless B. Under general conditions and slowly grows in size. Objectively: there C. Dry-air sterilization is rounded infiltrate with clear margins in the D. Processing with lysol mental region; it is slightly painfiul, the skin E. Processing with 0,1% solution of chloramine under it is very thin. Palpation in the vestibule of mouth reveals dense band going from the 168. A 50-year-old patient complains of his growth to the destroyed 33rd tooth. What is lower jaw teeth mobility. Objectively: the the most probable diagnosis? dentition is intact; the necks of the teeth are bared; the tooth crowns are tall; teeth mobility A. Odontogenic granuloma of the face is class 1-2. What kind of dental splint is the B. Odontogenic abscess of the mental region most advisable? C. Atheroma of the mental region D. Acute lymphadenitis the mental region A. Elbrecht splint E. Chronic lymphadenitis of the mental region B. Mamlock splint C. Full crown splint 164. A 43-year-old patient complains of mobi- D. Equator crown splint lity of all his teeth. Upon examination and E. Cap splint additional tests the following diagnosis is made: generalized periodontitis. What kind of 169. A patient complains of an increasing treatment does this patient need? new growth in the left sublingual area, whi- ch he first noticed 2 months ago. Some ti- A. Complex me after that he noticed this new growth to B. Prosthodontic secrete large amount of clear viscous fluid, C. Surgical which was colorless and tasteless; initially this D. Therapeutic caused the new growth to become smaller but E. Orthodontal subsequently it started to grow again. Objecti- vely: there is a rounded new growth sized 3x3 165. A 38-year-old female complains of cm in the left sublingual area, its consistency is experiencing pain in her lower lip for half soft and elastic, there is no pain. The mucosa a year, difficult eating and talking. The pati- above is thinned out, semitransparent, bluish ent sought medical help at place of residence, in shade. What is the most probable diagnosis? but the administered treatment was ineffecti- ve. Examination of the lower lip revealed A. Retention sublingual cyst a deep fissure in the centre which affected B. Hemangioma of sublingual area the vermilion border and partly the mucous C. Dermoid cyst of sublingual area membrane of lips. The fissure was covered D. Post-traumatic cyst of sublingual area with a brown crust, after its removal the fi- E. Pleomorphic adenoma of ssure started bleeding. The fissure edges are hyperkeratinized, infiltrated. Select the opti- 170. After extreme exposure to cold a 42-year- mal treatment tactics: old patient complains of headache in the left frontal lobe and the left upper jaw. Objecti- A. Fissure excision vely: the face is symmetrical; left nasal meatus B. Application of keratoplastic agents breathing is obstructed, and serous-purulent C. Irradiation with helium-neon laser discharge is being produced; palpation of the D. Application of corticosteroid ointment suborbital area and further along the mucogi- E. Application of antibiotic ointment ngival fold in the 24th, 25th teeth projecti- on reveals slight pain. Percussion of these 166. A 44-year-old male patient diagnosed with teeth is painless. The 24th tooth has a filli- chronic generalized periodontitis, class II, has ng. The alveolar process mucosa has no visi- had flap surgery of the lower jaw. What kind of ble changes. X-ray imaging shows decreased oral hygiene procedures are recommended for pneumatization of the left maxillary sinus. this patient during the first 2-3 days after the What is the provisional diagnosis? surgery? Крок 2 Stomatology (англомовний варiант, iноземнi студенти) 2015 рiк 21

A. Exacerbation of chronic odontogenic maxi- A. To file down the edges llary sinusitis B. To insert parapaulpar posts B. Acute periodontitis of the 24th C. To use extra hard material C. Exacerbation of chronic periodontitis of the D. To make retention furrows in the area of 24th enamel-dentin joining D. Acute rhinogenous maxillary sinusitis E. To shape a wide-angled slant of enamel E. Acute albuminous periostitis of the left maxilla 175. A 19-year-old young man complains of unaesthetical look of his teeth, nearly all of 171. A 8,5-year-old child is practically in good which have spots and defects; his teeth have health. There is a complaint of pain in the had such appearance since the moment of upper left tooth, due to it having been physi- teething. The patient was born and had been cally damaged 3 hours ago. Objectively: 1/2 of living until the age of 5 in the area, where the the 21st tooth crown is destroyed, the pulp is fluorine content of drinking water was 3,2 mg/l. significantly exposed, red, sharply painful and Objectively: the enamel of all his teeth is of bleeding when probed. Percussion of the 21st dull chalky shade with separate dark-brown tooth is sharply painful. Choose the optimal spots, and has multishaped defects with dense method of the 21st tooth treatment. margins, that are the most pronounced on the front upper teeth. What type of fluorosis does A. Vital amputation this patient have? B. Vital extirpation C. Non-vital amputation A. Erosive D. Non-vital extirpation B. Lined E. Biological method C. Patchy D. Chalky-speckled 172. A 34-year-old male patient complains of E. Destructive a cosmetic defect, a cavity on the vestibular surface in the cervical part of the 21 tooth. 176. To fill a medium depth carious cavity Objectively: the carious cavity is within the in the 37th tooth (Black class 2) of a 35- enamel, the floor and the walls are pigmented, year-old male patient a doctor has chosen a probing and percussion and painless. There is technique of layer-by-layer tooth restoration. no pain reaction to stimuli. What is the most What composite material should be applied for likely diagnosis? covering the carious cavity walls and floor to create the initial super adaptive layer? A. Chronic surface caries B. Acute surface caries A. Flowable C. Necrosis of dental hard tissues B. Condensable D. Acute median caries C. Macrofilled E. Chronic median caries D. Microhybrid E. Microfilled 173. A 6-year-old child complains of pain in the right submandibular region. A day before he 177. A 13-year-old girl has bleeding gums and complained of a sore throat. Objectively: the her front teeth are mobile. These symptoms child is in moderately severe condition, body have been observed during the last month. it is 37, 9oC. The face is asymmetric due to a Objectively: the gum mucosa in the area of dense tender infiltrate in the right submandi- the lower incisors and canines is swollen, bular region. The infiltrate is slightly mobi- hyperemic, bleeding when touched. There is le, the overlying skin is hyperemic. There is class I teeth mobility, periodontal pockets are fluctuation in the center. The teeth are intact. 3 mm deep. Orthopantomogram shows bone What is the most likely diagnosis? tissue destruction of the alveolar septums up to 1/3 of their height. Congestion of the lower A. Acute suppurative non-odontogenic jaw front teeth can be observed. Oral Hygi- submandibular lymphadenitis ene Index equals 4,2. What specialist must be B. Acute serous non-odontogenic submandi- consulted first? bular lymphadenitis C. Phlegmonous adenitis A. Orthodontist D. Acute suppurative odontogenic submandi- B. Neurologist bular lymphadenitis C. Endocrinologist E. Sialadenitis D. Hematologist E. Gastroenterologist 174. A dentist treats a 35-year-old patient from chronic deep caries of the 24th tooth that has 178. A 45-year-old patient complains of being mesial-occlusal-distal cavity. What should the unable to chew food due to loss of the lateral dentist do to prevent the cavity edges from chi- teeth. The rest of his teeth have mobility of the pping? 1-2 type. What construction is the most advi- sable for dental prosthetics? Крок 2 Stomatology (англомовний варiант, iноземнi студенти) 2015 рiк 22

A. Removable denture splint A. Combination of photopolymer material with B. Fixed dental bridge fiberglass framework C. Metal-based prosthesis B. Immediate prosthesis D. Clasp (bugel) dental prosthesis C. Adhesive dental bridge E. Removable laminar denture D. Porcelain-fused-to-metal whole piece dental bridge 179. A 65-year-old male patient complains of a E. Wire splint long-lasting pain in a mandibular tooth on the right. The pain increases on biting. The patient 183. A 65-year-old male patient complai- has a history of this tooth treatment for cari- ns of crepitation and clicking in both es. Objectively: the cavity on the masticatory temporomandibular joints; pain when movi- surface of the 36 tooth is filled, percussion ng his mandible to the right; ear noise; causes pain responce. Along the mucogingi- dry mouth; glossalgia. He has been using a val junction in the projection of the 36 tooth complete removable denture for the mandi- roots there is a fistula with purulent discharge. ble for 6 months. The patient has no history What is the most likely diagnosis? of rheumatosis. Objectively: the lower third of the face is shortened, mental fold is strongly A. Chronic granulating periodontitis pronounced, mouth corners are downturned, B. Radicular cyst of the mandible there are angular fissures and cracks. Palpation C. Exacerbation of chronic periodontitis reveals crepitation during TMJ moving. What D. Chronic fibrous periodontitis is the most likely diagnosis? E. Chronic granulomatous periodontitis A. Costen’s syndrome 180. During examination of an 8-year-old chi- B. Temporomandibular arthritis ld there were spots detected on the vesti- C. Temporomandibular arthrosis bular surface of the 11th, 12th, 21st and 22nd D. Temporomandibular dislocation teeth. The spots are pearly-white, glossy, pai- E. Temporomandibular osteoarthritis nless when probed, and gradually merge with non-changed enamel. Ultraviolet irradiation 184. A 45-year-old patient undergoes teeth revealed the spots to fluoresce blue. The child preparation. 15 minutes after anesthetization has been living up to the age of 3 in the region, with 4% solution of Ubistesin forte the pati- where fluorine content of drinking water was 2 ent developed hyperemia of skin, increased mg/l. What kind of non-carious lesion is it? heart rate, headache, syncope. Previously the patient had not exhibited such reaction to this A. Fluorosis, speckled form anesthetic. What complication occurred in the B. Fluorosis, lined form patient? C. Fluorosis, erosive form D. Local hypoplasia A. Arterial pressure rise E. Systemic hypoplasia B. Anaphylactic shock C. Syncope 181. A woman with complaints of restricted D. Collapse mouth opening has made an appointment wi- E. Myocardial infarction th a prosthodontic clinic. What muscles are responsible for the downwards mowement of 185. An accident victim with an injury of face the lower jaw? and the temporal region was diagnosed wi- th a fracture of the zygomatic arch. Which A. Mylohyoid muscle, digastric muscle, geni- processes of the skull bones are damaged? ohyoid muscle B. Masticatory muscle, medial pterygoid muscle A. Temporal process of the zygomatic bone and C. Temporal muscle, lateral pterygoid muscle, the zygomatic process of the temporal bone digastric muscle B. Zygomatic process of the frontal bone and D. Orbicular muscle of mouth, mylohyoid the zygomatic process of the temporal bone muscle C. Temporal process of the zygomatic bone and E. Geniohyoid muscle, medial pterygoid muscle the zygomatic process of the frontal bone D. Zygomatic process of the maxilla and the 182. A 45-year-old patient is diagnosed wi- zygomatic process of the temporal bone th generalized periodontitis; there is class E. Zygomatic process of the maxilla and the II mobility of the 13th, 12th, 21st, 22nd, zygomatic process of the frontal bone 23rd teeth. The patient has made an appoi- ntment with dental clinic to make a prostesis 186. A 20-year-old male patient complains of after the extraction of the 11th tooth. What spontaneous pain in the 24 tooth, which arose prosthodontic construction is the most advi- suddenly and persists for about 15 minutes. sable for combined therapy of generalized Objectively: the distal surface of the 24 tooth periodontitis in this case? exhibits a deep carious cavity with overhangi- ng walls. The cavity is filled with light softened dentin and communicates with the tooth cavity. The cold stimulus causes acute, slowly abati- ng pain. Percussion causes no pain response. Select the best method of treatment: Крок 2 Stomatology (англомовний варiант, iноземнi студенти) 2015 рiк 23

A. Vital extirpation this case? B. Vital amputation C. Biological method A. Cast removable splint D. Devital amputation B. Kurlyandsky splint bar E. Devital extirpation C. Cap splint D. Portion crown 187. A 25-year-old patient complains of pain E. Half-ring splint when biting on the 15 tooth. The pain arose two days ago, has a constant aching nature 191. A 7-year-old child has to undergo plastic and increased significantly over the last day. surgery of the upper lip frenulum. What Objectively: the crown of the 15 tooth is gray, operation should be performed to lengthen the medial contact surface exhibits a deep cari- the frenulum of the upper lip? ous cavity communicating with the tooth cavi- ty. Percussion causes acute pain, the gingival A. Dieffenbach grafting mucosa in the projection of the 25 tooth root B. Thiersch grafting with local flaps apex is hyperemic. The regional lymph node is C. Relaxing incisions of the mucosa tender. Radiograph shows an ill-defined zone D. Grafting with a pedicle flap of periapical bone destruction. What is the E. Szymanowsky grafting with local flaps most likely diagnosis? 192. A 12-year-old boy is in the dental offi- A. Exacerbation of chronic periodontitis ce to complete treatment of the 36th tooth B. Acute serous periodontitis chronic periodontitis exacerbation. There are C. Acute suppurative periodontitis no complaints. Objectively: occlusive dressi- D. Chronic granulating periodontitis ng of the 36th tooth is intact; tooth percussi- E. Acute serous periodontitis, intoxication stage on is painless; there are no pathologies of the mucosa surrounding the 36th; mucosa palpati- 188. A 27-year-old patient has been referred on is painless. What material is the most advi- by a prosthodontist for endodontic treatment sable for root canal filling in this case? of the 45th tooth. Objectively: the 45th tooth crown is destroyed; the lateral surface of the A. Sealer with gutta-percha post tongue and the buccal mucosa have patches B. Zinc oxide eugenol paste of greyish macerated epithelium slightly risi- C. Resorcinol-formalin paste ng above the mucosa surface at the points of D. Calcium-containing paste direct contact with the 45th tooth. The uvula E. Phosphate cement and palatal bars are stagnant-red in colour; hard palate has papulae surrounded with red 193. A patient with complaints of toothache in margin and covered in greyish epithelium. The the left upper jaw has made an appointment submandibular, cervical, supraclavicular, and with a dental clinic. He was diagnosed with subclavicular lymph nodes are enlarged and chronic periodontitis of the 24th tooth. What painless. What is the provisional diagnosis? kind of anaesthesia is necessary for painless extraction of the 24th tooth? A. Secondary syphilis B. Chronic recurrent aphthous stomatitis A. Infraorbital and palatinal anaesthesia C. Lupus erythematosus, patch stage B. Tuberal and palatinal anaesthesia D. Soft leukoplakia (leucoplakia mollis) C. Infraorbital and incisor anaesthesia E. Lichen ruber planus D. Tuberal and incisor anaesthesia E. Surface and tuberal anaesthesia 189. A 20-year-old male patient with insulin- dependent diabetes mellitus has been given 194. A 60-year-old patient needs a partial anesthesia with Ultracain DS-forte for surgi- removable denture for the upper jaw. The cal oral sanitation. What complications may central occlusion has been determined. What develop in this case? is the next stage of prosthetics? A. Abnormal levels of blood glucose A. Checking the wax denture construction B. Increased blood pressure B. Try-in of the denture C. Clonus C. Fabrication of plaster models D. Atopic dermatitis D. Relining the denture E. Toxic shock E. Taking impressions 190. A 43-year-old patient complains of mobi- 195. A 12-year-old boy, due to trauma of lity and significant neck exposure of the lower the 44th and 45th teeth area, suffers from front teeth. Objectively: the gums in the regi- the following: pathologic displacement of the on of the 44, 43, 42, 41, 31, 32, 33, 34 teeth mandibular alveolar process; rupture of the are pale and cyanotic, non-bleeding. The 42, alveolar process mucosa. What additional 41, 31, 32 teeth exhibit the I-II grade mobility. examination is necessary to specify the di- The overcrowding of the 42, 41, 31, 32 teeth is agnosis? present. The necks of the 42, 41, 31, 32 teeth are exposed by 1/2 of the root length, the necks of the 43, 33 teeth are exposed by 1/4. What ki- nd of dental prosthesis should be applied in Крок 2 Stomatology (англомовний варiант, iноземнi студенти) 2015 рiк 24

A. X-ray radiography of mandibula in frontal is destroyed with cariosity. X-ray image of and lateral projections the paranasal sinuses shows the left one to B. X-ray radiography of cranium in axillary have veil-like shading with clear dome-shaped projection margin. X-ray image of the crown of the 25th C. X-ray radiography of mandibula in frontal tooth shows absence of the periodontal fissure and Parma projection at the the palatal root apex. What is the most D. Tomography of mandibula probable diagnosis? E. - A. Radicular cyst that invaded in the maxillary 196. A 5-year-old shild has sustained a trauma. sinus Objectively: the crowns of the 51st and 61st B. Chronic rhinogenous maxillary sinusitis teeth are shorter then their adjacent ones. The C. Chronic odontogenic maxillary sinusitis mucosa surrounding the 51st and 61st teeth is D. Maxillary sinus mucosal cyst hyperemic and swollen. X-ray imaging shows E. Maxillary cancer the periodontal fissure to be absent in the api- cal area of the 51st and 61st roots, 51st and 199. A 26 year-old female patient suffers from 61st roots apexes are submerged in the spongy bleeding gums. For 3 years she has been bone of the body of the lower jaw. Choose the suffering from pancreatic diabetes. Objecti- optimal treatment. vely: gingival papillas of all her teeth are hyperemic, swollen and overlap with the teeth A. Extraction of the 51st and 61st teeth crowns by 1/3 of the crowns height. What B. Splinting of teeth method of examination must be conducted for C. Regular medical check-up differentiated diagnostics? D. Anti-inflammatory therapy E. Resection of the 51st and 61st teeth and their A. X-ray radiography splinting B. Schiller-Pysarev iodine test C. Capillary fragility test 197. A 10-year-old child complains of pain and D. Reoparodontography (Impedance swelling in the left submandibular region; eati- phlebography of periodontium) ng causes pain. Objectively: the mouth can be E. Stomatoscopy fully opened, the mucosa of the torus mandi- bularis is hyperemic, excretory duct produces 200. Parents of a 3-year-old child took the child pus. X-ray imaging reveals an oval shade in to a pedodontist for oral sanitation. The child the left submandibular area. What is the most has cerebral palsy. Objectively: there is multi- probable diagnosis? ple dental caries, gingival mucosa is hyperemic, oral hygiene is inadequate. What method of A. Calculous sialadenitis of submandibular dental health care may be appropriate in this salivary gland case? B. Mandibular osteoma C. Foreign body A. Under general anesthesia D. Chronic lymphadenitis B. Under local anesthesia injection E. Chronic sialadenitis C. Under local application anesthesia D. Without anesthesia 198. A patient complains of the alveolar E. Giving sedatives process deformation of the left upper jaw. Objectively: the crown of the 25th tooth

INSTRUCTIONAL BOOK

Ministry of public health of Ukraine (MPH of Ukraine) Department of human recources policy, education and science Testing Board

TEST ITEMS FOR LICENSING EXAMINATION: KROK 2. STOMATOLOGY.

Kyiv. Testing Board. (English language).

Approved to print 18.03./№24. Paper size 60х84 1/8 Offset paper. Typeface. Times New Roman Cyr. Offset print. Conditional print pages 24. Accounting publishing pages 28. Issue. 333 copies.

List of abbreviations

A/G Albumin/globulin ratio HR Heart rate A-ANON Alcoholics anonymous IDDM Insulin dependent diabetes mellitus ACT Abdominal computed tomography IFA Immunofluorescence assay ADP Adenosine diphosphate IHD Ischemic heart disease ALT Alanin aminotranspherase IU International unit AMP Adenosine monophosphate LDH Lactate dehydrogenase AP Action potential MSEC Medical and sanitary expert committee ARF Acute renal failure NAD Nicotine amide adenine dinucleotide AST Aspartat aminotranspherase NADPH Nicotine amide adenine dinucleotide phosphate restored ATP Adenosine triphosphate NIDDM Non-Insulin dependent diabetes mellitus BP Blood pressure PAC Polyunsaturated aromatic carbohydrates bpm Beats per minute PAS Periodic acid & shiff reaction

C.I. Color Index pCO2 CO2 partial pressure

CBC Complete blood count pO2 CO2 partial pressure CHF Chronic heart failure pm Per minute CT Computer tomography Ps Pulse rate DIC Disseminated intravascular coagualtion r Roentgen DCC Doctoral controlling committee RBC Red blood count DM-2 Non-Insulin dependent diabetes mellitus RDHA Reverse direct hemagglutination assay DTP Anti diphtheria-tetanus vaccine Rh Rhesus ECG Electrocardiogram (R)CFT Reiter's complement fixation test ESR Erythrocyte sedimentation rate RIHA Reverse indirect hemagglutination assay FC Function class RNA Ribonucleic acid FAD Flavin adenine dinucleotide RR Respiratory rate

FADH2 Flavin adenine dinucleotide restored S1 Heart sound 1 FEGDS Fibro-esphago-gastro-duodenoscopy S2 Heart sound 2

FMNH2 Flavin mononucleotide restored TU Tuberculin unit GIT U Unit Gy Gray USI Ultrasound investigation GMP Guanosine monophosphate V/f Vision field Hb Hemoglobin WBC White blood count HbA1c Glycosylated hemoglobin X-ray Roentgenogram Hct Hematocrit HIV Human immunodeficiency virus